202
Colen Publishing Colen Publishing www colenpublishing com ©™ ©™ www.colenpublishing.com

Comprehensive Neurology Board Review Flash cards

Embed Size (px)

DESCRIPTION

High yield comprehensive neuroscience flash cards with board style question and referenced explained answers. This review is all inclusive of all aspects of the neurology and psychiatry boards; includes neurology, neurobehavioral and psychiatry, child neurology, pathology, anatomy, critical care, pharmacology, radiology and biology.Ideal for use when studying advanced neuroscience as a student or resident. Faculty would benefit using these flash cards as a quick refresher of high-yield topics in neurology.Carry 10-15 cards in your pocket and study from these cards to utilize your time spent while waiting for an elevator, lunch line, or on the ward.Please visit our website: www.colenpublishing.com for more information.

Citation preview

Page 1: Comprehensive Neurology Board Review Flash cards

Colen PublishingColen Publishing

www colenpublishing com©™©™

www.colenpublishing.com

Page 2: Comprehensive Neurology Board Review Flash cards

Colen PublishingColen Publishing, L.L.C.PO Box 36536Grosse Pointe Woods, MI 48236Author and Editor: Chaim B. Colen, M.D., Ph.D.Editorial Assistant: Roxanne E. Colen, PA-C

Colen Publishing

Editorial Assistant: Roxanne E. Colen, PA C

COPYRIGHT © 2008 by Colen Publishing, L.L.C. This book, including all parts thereof, is legally protected by copyright. Any use, exploitation, or commercialization outside the narrow limits set by copyright legislation without the author’s consent if illegal and liable to prosecution. This applies in particular to photostat reproduction, copying, mimeographing or duplication of any kind, translating, preparation of microfilms, and electronic data processing and storage.Some of the product names, patents, and registered designs referred to in this book are in fact registered trademarks or proprietary names even though specific reference to this fact is not always made in the text. Therefore, the appearance of a name without designation as proprietary is not to be construed as a representation by the publisher that it is in the public domain. Permissions may be sought directly from Colen Publishing, L.L.C. by writing to the above address.Printed in ChinaISBN

PakPal:9781935345091 1935345095 Colen FlashReview: NPath 2009ISBN

Colen Flash-Review: NeurologyVolume 1: 0-9788502-4-6Volume 2: 0-9788502-5-42 Volume Set: 0-9788502-9-7

Note: Knowledge in medicine is constantly changing. The author has consulted sources believed to be reliable in the effort to provide information that is complete and in accord with the standards at the time of publication. However, in view of the possibility of human error by the author in preparation of this work, warrants that the information contained herein is in every respect accurate and complete, and that the author is not responsible for any errors or omissions or for the results obtained from use of such information. The reader is advised to confirm the information contained herein with other sources This is especially important in connection with new or infrequently used drugs In such instances the product

9781935345107 1935345109 Colen FlashReview: Nlgy 20099781935345114 1935345117 Colen FlashReview: NAn 20099781935345121 1935345125 Colen FlashReview: NCC 20099781935345138 1935345133 Colen FlashReview: NRad 20099781935345145 1935345141 Colen FlashReview: NBio 20099781935345152 193534515X Colen FlashReview: NBeh 2009

©™©™

to confirm the information contained herein with other sources. This is especially important in connection with new or infrequently used drugs. In such instances, the product information sheet included in the package with each drug should be reviewed.

Page 3: Comprehensive Neurology Board Review Flash cards

GlossaryGlossary

COPYRIGHT-------------------------------------------------- 1PREFACE------------------------------------------------------ 1PREFACE------------------------------------------------------ 1HOW TO USE THIS CARD REVIEW-------------------- 1CONTRIBUTORS-------------------------------------------- 4GLOSSARY--------------------------------------------------- 1NEUROSURGERY------------------------------------------ 110NEUROLOGY 86NEUROLOGY ------------------------------------------------ 86NEUROPATHOLOGY-------------------------------------- 238NEUROANATOMY----------------------------------------- 57NEUROCRITICAL CARE---------------------------------- 80NEURORADIOLOGY--------------------------------------- 73NEUROBIOLOGY 64

©™©™NEUROBIOLOGY------------------------------------------- 64 BONUS BIOSTATISTICS---------------------------------- 6

Page 4: Comprehensive Neurology Board Review Flash cards

PrefacePreface• The idea to undertake such a large Flashcard review spawned from watching my wife Roxanne

study for her Physician Assistant Boards. Diligently every day she would create a set of 7-10 flashcards from her study material that she would take with her to work. Later on, when I was y ,studying for my written Neurosurgery Board examination, I gleaned information from various texts and other study guides and wrote down the most relevant material on cards for quick review while at work. It was amazing how much time during the day would be available to review these cards. If there was a delay in a OR case, a long lunch-line, a traffic jam (especially the i94 on a Friday afternoon) or waiting for my wife at her OB/GYN appointment -these little cards were specially handy. Always ambitious in life, the thought of giving this study tool to the busy neurosurgery resident was captivating. My expectation is to enable the resident with a quick yet informative review of basic neuroscience principles. With positive encouragement from my fellow residents on the 1st edition, I cautiously proceed here with updating information, adding new images, improved illustrations and clarification of neuroscience concepts. May this endeavor

©™©™serve to better our wonderful science inherited through the legacy of Harvey Cushing, Neurosurgery.

Chaim September 9, 2008

Page 5: Comprehensive Neurology Board Review Flash cards

The Colen Flash ReviewThe Colen Flash-Review

Author and EditorChaim B. Colen, M.D., Ph.D.Chaim B. Colen, M.D., Ph.D.Department of Neurological SurgeryWayne State University School of MedicineDetroit, Michigan

Assistant EditorRoxanne E. Colen, M.S., PA-CColen Publishing, LLCGrosse Pointe, Michigan

©™©™

Page 6: Comprehensive Neurology Board Review Flash cards

AcknowledgementsAcknowledgementsI would like to give thanks to a great many wonderful persons whose efforts, although not inscribed in

these cards, were instrumental in making this monumental task possible. One exceptional individual to whom I owe special thanks is my mother in-in-law, Colleen Johns, who babysat my daughter Emily and son Joshua for hours on end while my wife and I toiled through hundreds of pages of various textbooks and journal articleshours on end, while my wife and I toiled through hundreds of pages of various textbooks and journal articles, formatted questions, and drew computer illustrations. To my daughter Emily Rivka, who incessantly tugged at my pants trying to get my attention to the squirrel in our backyard ;and that big bright smile from my son Joshua that continually sent me optimism. To Mahmoud and Abhi who spent hours at my home assisting with typing, researching and editing; Naomi whose positive attitude in life is exceptionally brightening and uplifted the group’s 2 am brainstorming sessions when I still had to wake up early to work the next day, all the pathologists, especially Doha, who assisted in taking photographs, Dr. William Kupsky, for allowing us access to his collection of unique neuropathology, and to all the medical students especially Kristyn, whose hard work is admirable. There are those whose names are not here but did assist in some way, thank you. I am forever indebted to my training program, the Wayne State University neurosurgery program, my Chairman Dr. Murali Guthikonda, and Associate Chairman Dr. Setti S. Rengachary whose moral support over the last five years has kept me on this educational drive. For this second edition, there were fellow residents that gave me input and new insight that has helped to improve this edition over the first.

To my parents Joseph and Leila, educators of true dedicated quality, and to whom I owe my homeschooling education and self-motivation. Lastly to my wife Roxanne, whose patience with my

biti k b d i©™©™

ambitiousness knows no boundaries. Thank you All,Chaim September 9, 2008

Page 7: Comprehensive Neurology Board Review Flash cards

How to use this Flashcard reviewHow to use this Flashcard review

• These cards are intended to cover most of the aspects of the Neurosurgery Board Examination. They are not a COMPLETE g y yreview and therefore they are not intended to replace textbooks. We would advise using these cards during the last couple of weeks before your board exam except for the pathology section which you should go through all year to better remember the photographs in it (heavily encountered during the boards!). BOARD FAVORITE( y g )questions are of extreme importance and most likely to bump into during the boards, so make you sure you know how to answer them right.

• Good luck!• Chaim B Colen M D Ph D

©™©™• Chaim B. Colen, M.D., Ph.D.

Page 8: Comprehensive Neurology Board Review Flash cards

Faculty ReviewersFaculty Reviewers

Murali Guthikonda, MD Professor and Chairman D t t f N l i l SDepartment of Neurological SurgeryWayne State University School of MedicineDetroit, Michigan

Setti Rengachary, MD Associate Chairman D t t f N l i l SDepartment of Neurological SurgeryWayne State University School of MedicineDetroit, Michigan

William, J. Kupsky, MDDepartment of Neuropathology W St t U i it S h l f M di i

©™©™Wayne State University School of MedicineDetroit, Michigan

Page 9: Comprehensive Neurology Board Review Flash cards

Forward• With ever increasing scope and complexity of knowledge base, the current

day trainee or practitioner of neurosurgery finds it difficult to keep up with the explosion of neurosurgical information. This is compounded by a

Forward

healthy growth in specialization in various branches of neurosurgery.• Chaim has made an attempt to make life simpler by incorporating small

quanta of knowledge on flashcards accompanied by clear and simple illustrations. The user may review as few or as many cards as his/her time will allow. Although not meant to be substitutes for standard comprehensive t t d tl th d h l t f h th i f ti l d ftexts and atlases, these cards help to refresh the information learned from the bedside, operating room and standard books. Each card represents a mini-examination with instant access to appropriate answers.

• This is a fun way to recall neurosurgical information especially before an upcoming test.

©™©™Setti S. Rengachary, M.D.Department of Neurological Surgery

Page 10: Comprehensive Neurology Board Review Flash cards

Physician Contributing AuthorsPhysician Contributing Authors

Mahmoud Rayes, MDRivka R. Colen, MDDepartment of Neurological Surgery WSU School of Medicine

Erika Peterson, MDUT Southwestern,

,Department of RadiologyThe Massachusetts General HospitalHarvard Medical SchoolBoston, Massachusetts

Doha Itani MD ,Department of Neurological Surgery Dallas, Texas

Doha Itani, MDDepartment of PathologyWSU School of MedicineDetroit, Michigan

©™©™

Page 11: Comprehensive Neurology Board Review Flash cards

Contributing Medical StudentsContributing Medical Students

Darmafall, KristynWayne State UniversitySchool of Medicine

Galinato, AnthonyWayne State UniversitySchool of Medicine

Larson, SarahWayne State UniversitySchool of Medicine

Class of 2012

Davis, Naomi Wayne State UniversitySchool of Medicine Class of 2011

Dub Larissa

Class of 2012

Gotlib, DorothyWayne State UniversitySchool of MedicineClass of 2009

Kozma Bonita

Class of 2012

Martinez, DerekWayne State UniversitySchool of MedicineClass of 2011

Matthew SmithDub, LarissaWayne State UniversitySchool of MedicineClass of 2012

Faulkiner, RodneyWayne State UniversitySchool of Medicine

Kozma, BonitaWayne State UniversitySchool of MedicineClass of 2008

Lai, Christopher Wayne State UniversitySchool of Medicine

Matthew SmithWayne State UniversitySchool of MedicineClass of 2011

Matto, ShereenWayne State UniversitySchool of Medicine

©™©™Class of 2012 Class of 2010 Class of 2012

Page 12: Comprehensive Neurology Board Review Flash cards

Contributing UndergraduatesContributing Undergraduates

Jeffrey P. KallasWayne State UniversityClass of 2010

Abhinav KrishnanWayne State UniversityClass of 2010

Peter PaximadisWayne State UniversityClass of 2008

©™©™

Page 13: Comprehensive Neurology Board Review Flash cards

NeuropathologyQ?• This cerebellar specimen is

consistent with:

NeuropathologyQ?

A. Lewy bodyB. Neuritic plaqueC. Kuru plaqueD. Hirano body

©™©™

2

Page 14: Comprehensive Neurology Board Review Flash cards

Neuropathology SectionNeuropathologyA Neuropathology SectionNeuropathologyA.• The correct answer is C, Kuru plaque.• Pathology of variant Creutzfeldt-Jakob

Kuru Plaque

disease – Kuru demonstrating a “kuru plaque” in cerebellum.

• Accumulation of prion protein PrP in the form of amyloid plaques occurs in some forms of prion disease.p

• Hirano bodies (rod-shaped eosin inclusions) are seen in Alzheimer’s disease.

PAS

©™©™Cerebellum: granular cell layerLiberski PP. Amyloid plaques in transmissible spongiform encephalopathies (prion diseases). Folia

Neuropathol. 2004;42 Suppl B:109-19. Review.

Cerebellar Infection, Prion, Creutzfeldt-Jakob disease Kuru amyloidJakob disease, Kuru, amyloid

Page 15: Comprehensive Neurology Board Review Flash cards

NeuropathologyQ?• This brain specimen is most

consistent with:

NeuropathologyQ?

A. Lewy bodyB. Neuritic plaqueC. Kuru plaqueD. Hirano body

©™©™

3

Page 16: Comprehensive Neurology Board Review Flash cards

NeuropathologyA NeuropathologyA.• The correct answer is D, Hirano body.• Hirano bodies (rod-shaped eosin

Hirano body

BOARD FAVORITE!

inclusions of mostly actin filaments) are seen in Alzheimer’s disease.

• Accumulation of prion protein PrP in the form of amyloid plaques occurs in the pathology of variant Creutzfeldt-p gyJakob disease – Kuru demonstrates “kuru plaques” in the cerebellum.

• Lewy bodies are seen in Parkinson’s disease and diffuse Lewy body disease These are formed from α- Section of hippocampus

©™©™

disease. These are formed from α-synuclein.

Kuljis RO: Modular corticocerebral pathology in Alzheimer's disease. In: Mangone CA, Allegri RF, Ariza, eds. Dementia: A Multidisciplinary Approach. 1997: 143-55.

Degenerative disease, Alzheimer’s disease Hirano bodiesdisease, Hirano bodies

Page 17: Comprehensive Neurology Board Review Flash cards

NeuropathologyQ?• What is the most likely diagnosis seen

in this fundoscopic examination?

NeuropathologyQ?

A. Drusen bodiesB. Retinopathy of diabetesC. Retinal hemorrhageD. Pseudopapilledema

Used with permission from Handbook of Neurosurgery andNeurology in Pediatrics; By Chaim B. Colen, MD, PhD.

©™©™

9

Page 18: Comprehensive Neurology Board Review Flash cards

NeuropathologyA NeuropathologyA.• The correct answer is C, retinal hemorrhages.• Shaken baby syndrome was described by Caffey as a clinical triad: subdural

hematomas, subarachnoid hemorrhages, and retinal hemorrhages.• "Shaken baby syndrome" (SBS) results in intracranial and intraocular hemorrhages

with no evidence of external trauma. The cause of these injuries is vigorous shaking of an infant being held by the chest, shoulders, or extremities.

• Physicians are mandated to report cases of child abuse elder abuse sexual abuse Physicians are mandated to report cases of child abuse, elder abuse, sexual abuse, domestic violence, and assault. Physicians are also required to notify the authorities if anyone has a specific plan to commit suicide or homicide.

• Drusen bodies aka “pseudopapilledema” are colloid or hyaline bodies lying beneath the retinal pigment epithelium. They may occur either secondary to changes in the choroid that affect the pigment epithelium or as an autosomal dominant disorder of

©™©™choroid that affect the pigment epithelium or as an autosomal dominant disorder of the retinal pigment epithelium.

Martin HA, Woodson A, Christian CW, Helfaer MA, Raghupathi R, Huh JW. Shaken baby syndrome. Crit Care Nurs Clin North Am. 2006 Sep;18(3):279-86. Review.

Shaken baby syndrome Pediatrics RetinalShaken baby syndrome, Pediatrics, Retinal hemorrhages, subdural hematomas,

subarachnoid hemorrhages

Page 19: Comprehensive Neurology Board Review Flash cards

NeuropathologyQ?• Which pairing is most accurate for the localization of medulloblastomas in the pediatric

and adult population?

NeuropathologyQ?

A. Children : Midline, Adults : MidlineB. Children : Lateral, Adults : LateralC. Children : Lateral, Adults : MidlineD. Children : Midline, Adults : Lateral

©™©™

54

Page 20: Comprehensive Neurology Board Review Flash cards

NeuropathologyA NeuropathologyA.• The correct answer is D, children :

midline, adults: lateral

BOARD FAVORITE!

Homer-Wright rosette –(pseudorosette) radially arranged nuclei surrounding central fib ill t i l ( d ) hi t th l i l MRI b i t t h d

©™©™fibrillary material (red arrow); histopathological features of medulloblastoma.

MRI brain -contrast enhanced sagittal image showing the midline location of the tumor in a pediatric patient.

Neoplasm, Cerebellum, PNET, medulloblastoma, plocation of medulloblastoma, Homer-Wright

rosette

Page 21: Comprehensive Neurology Board Review Flash cards

NeuropathologyQ?• Compared to pilocytic

astrocytoma:

NeuropathologyQ?

A. This tumor has a better prognosis

B. This tumor has a similar prognosis.

C. This tumor has a worse C. This tumor has a worse prognosis

D. Prognosis is irrelevant

©™©™

114

Page 22: Comprehensive Neurology Board Review Flash cards

NeuropathologyA NeuropathologyA.• The correct answer is C, this tumor has a worse prognosis..• Gemistocytic astrocytoma is WHO grade II and is distinctive in its appearance, having

t t ith i hili t l ith “ t ff d” t ll t

BOARD FAVORITE!

astrocytes with generous eosinophilic cytoplasm with a “stuffed” stellate appearance. • Low-grade astrocytomas with a significant fraction of gemistocytes progress more rapidly

and typically carry a p53 mutation.Secondary GBMGemistocytic astrocytoma

Secondary GBMProgenitor cell

pRB loss, NK4α increase, PTEN, loss

PDGFR increase

P53 loss

Grade IIastro

Grade IIIastro

©™©™Krouwer HG, Davis RL, Silver P, Prados M. Gemistocytic astrocytomas: a reappraisal. J Neurosurg. 1991 Mar;74(3):399-406.

p oss, α c ease, , oss

neoplasm, astrocytoma, gemistocytic astrocytoma, WHO grade III

Page 23: Comprehensive Neurology Board Review Flash cards

NeuropathologyQ?• This muscle specimen

(Gomori stain) is consistent ith hi h di d ?

NeuropathologyQ?

with which disorder?A. Myoclonic seizuresB. Disuse atrophyC. Steroid atrophyD Denervation atrophyD. Denervation atrophy

©™©™

138

Page 24: Comprehensive Neurology Board Review Flash cards

NeuropathologyA NeuropathologyA.• The correct answer is A, myoclonic

seizures.• Mitochondrial disorders (e g

BOARD FAVORITE!

• Mitochondrial disorders (e.g. MELAS, MERRF) clinically are associated with seizures and muscle biopsy shows ragged-red fibers and rimmed vacuoles.

• Type 1 fiber atrophy- think muscle di ! t i d t h d disease! -myotonic dystrophy and congenital myopathy

• Type 2 muscle atrophy- think neurogenic or muscle disease! –denervation, disuse, myasthenia gravis, steroids and paraneoplastic

RAGGED RED FIBERS RIMMED VACUOLE

©™©™syndrome (BOARD FAVORITE!)

Fukuhara N, Tokiguchi S, Shirakawa K, Tsubaki T. Myoclonus epilepsy associated with ragged-red fibres (mitochondrial abnormalities ): disease entity or a syndrome? Light-and electron-microscopic studies of two cases and review of literature. J Neurol Sci. 1980 Jul;47(1):117-33.

Muscle, Mitochondrial disease, seizures, ragged-red fibers, rimmed vacuoles, type 1 fiber atrophy,red fibers, rimmed vacuoles, type 1 fiber atrophy,

type 2 fiber atrophy

Page 25: Comprehensive Neurology Board Review Flash cards

NeuropathologyQ?• This peripheral nerve biopsy is

consistent with:A M asthenia gra is

NeuropathologyQ?

A. Myasthenia gravisB. Onion bulb formationC. Wallerian degenerationD. Neurofibroma

©™©™

139

Page 26: Comprehensive Neurology Board Review Flash cards

NeuropathologyA NeuropathologyA.• The correct answer is B, onion bulb

formation.• Onion bulb formation occurs

BOARD FAVORITE!

• Onion bulb formation occurs after repeated cycles of demyelination and remyelination result in a thick layer of abnormal myelin around the peripheral axons. These changes cause what is referred to as an onion bulb referred to as an onion bulb appearance.

• May be seen in a few disorders: Charcot-Marie-Tooth (CMT) and Refsum’s disease (phytanic acid disorder).

ONION BULBSONION BULB (electron microscopy)

©™©™Bornemann A, Hansen FJ, Schmalbruch H: Nerve and muscle biopsy in a case of hereditary motor and sensory neuropathy type III with basal lamina onion bulbs. Neuropathol Appl Neurobiol 1996 Feb; 22(1): 77-81.

ONION BULBS microscopy)

Peripheral nerve, Onion bulb formation, myelin, Charcot-Marie-Tooth, Refsum’s disease

Page 27: Comprehensive Neurology Board Review Flash cards

NeuropathologyQ?• Symptoms of a patient with this

diagnosis might be alleviated with hi h f th f ll i d

NeuropathologyQ?

which of the following drugs:A. Retroviral drugB. Anticholinesterase inhibitorC. Beta-interferonD TrimethoprimD. Trimethoprim-

sulfamethoxazole

©™©™

140

Page 28: Comprehensive Neurology Board Review Flash cards

NeuropathologyA NeuropathologyA.• The correct answer is A,

anticholinesterase inhibitor.Mi h l id A l id l• Microscopy shows an amyloid plaque and neurofibrillary tangles that would be consistent with the diagnosis of Alzheimer’s disease.

• Donepezil is a synthetic noncovalent reversible inhibitor

Neurofibrillary tangle

Amyloid plaque

noncovalent reversible inhibitor of acetylcholinesterase (AChE) for the treatment of mild to moderate dementia associated with Alzheimer's disease.

• Other drugs include galantamine ©™©™

Other drugs include galantamine and rivastigmine.

Kuljis RO: Modular corticocerebral pathology in Alzheimer's disease. In: Mangone CA, Allegri RF, Ariza, eds. Dementia: A Multidisciplinary Approach. 1997: 143-55.

Degenerative disease, Dementia, Alzheimer’s disease, neurofibrillary tangles, amyloid plaque,disease, neurofibrillary tangles, amyloid plaque,

Donepezil, Acetylcholinesterase inhibitor

Page 29: Comprehensive Neurology Board Review Flash cards

NeuropathologyQ?• This brain specimen is most

consistent with:

NeuropathologyQ?

A. Alzheimer’s diseaseB. Parkinson’s diseaseC. Pick’s diseaseD. Wilson’s disease

©™©™

142

Page 30: Comprehensive Neurology Board Review Flash cards

NeuropathologyA NeuropathologyA.• The correct answer is B,

Parkinson’s disease.

BOARD FAVORITE!

• Microscopy shows melanophagesand neuronal cell loss in the substantia nigra pars compacta.

• Lewy bodies are also seen. These are formed from α-synuclein.

• Pick bodies are seen in Pick’s disease using silver stain.

NEURONAL LOSS LEWY BODY

©™©™Hodaie M, Neimat JS, Lozano AM. The dopaminergic nigrostriatal system and Parkinson's disease: molecular events in development, disease, and cell death, and new therapeutic strategies. Neurosurgery. 2007 Jan;60(1):17-28; discussion 28-30.

Degenerative disease, Movement disorder, Parkinson’s disease, melanophages, lewy bodies,

alpha-synuclein

Page 31: Comprehensive Neurology Board Review Flash cards

NeuropathologyQ?• This neonatal brain specimen is

consistent with which hemorrhage grade?

NeuropathologyQ?

A. Grade IB. Grade IIC. Grade IIID. Grade IV

©™©™

148

Page 32: Comprehensive Neurology Board Review Flash cards

NeuropathologyA NeuropathologyA.• The correct answer is C, grade III.• This gross specimen demonstrates germinal matrix

hemorrhage (GMH) and intraventricular hemorrhage IVH and hydrocephalus

BOARD FAVORITE!

hemorrhage (GMH) and intraventricular hemorrhage (IVH) and hydrocephalus which are the most common and most important neurologic injuries in preterm neonates.

• Premature infants lack the ability to autoregulate the cerebral blood pressure. Fluctuations in cerebral bl d d fl t th i iti

GMH

blood pressure and flow can rupture the primitive germinal matrix vessels or lead to infarction of the metabolically active germinal matrix. Neurologic sequelae, include cerebral palsy, mental retardation, and seizures. GRADE 3

Grading System

©™©™Levy ML, Masri LS, McComb JG: Outcome for preterm infants with germinal matrix hemorrhage and progressive hydrocephalus. Neurosurgery 1997 Nov; 41(5): 1111-7; discussion 1117-8.

Grade 1 - confined GMHGrade 2 - IVH without hydrocephalus.Grade 3 - IVH with associated hydrocephalusGrade 4 - Intraparenchymal hemorrhage

Vasculature, hemorrhage, hemorrhage grading, germinal matrix hemorrhage intraventriculargerminal matrix hemorrhage, intraventricular

hemorrhage, Pediatrics

Page 33: Comprehensive Neurology Board Review Flash cards

NeuropathologyQ?• This pathology is most likely due

to:

NeuropathologyQ?

A. Vitamin A toxicityB. Cyanocobalamine

deficiency C. Thiamine deficiencyD Vitamin B deficiencyD. Vitamin B6 deficiencyE. Alcohol intake

©™©™

149

Page 34: Comprehensive Neurology Board Review Flash cards

NeuropathologyA NeuropathologyA.• The correct answer is C, thiamine

deficiency.

BOARD FAVORITE!

• Thiamine deficiency is seen in chronic malnourished alcoholics. Note the old infarction that has occurred in the mamillary bodies.

• Wernicke-Korsakoff’s syndrome –

Old infarction

ysubacute amnesia seen in damage to the mamillary bodies and may be caused by thiamine deficiency and may be reversible with the administration of thiamine.

©™©™Torvik A. Topographic distribution and severity of brain lesions in Wernicke's encephalopathy. Clin Neuropathol. 1987 Jan-Feb;6.

Nutritional deficiency, Thiamine deficiency, Wernicke-Korsakoff syndrome, mamillary bodies

Page 35: Comprehensive Neurology Board Review Flash cards

NeuropathologyQ?• This brain specimen is most

consistent with which disease?

NeuropathologyQ?

A. Wilson’s (hepatolenticular degeneration)

B. Alzheimer’sC. Pick’sD Parkinson’sD. Parkinson sE. Huntington’s

©™©™

151

Page 36: Comprehensive Neurology Board Review Flash cards

NeuropathologyA NeuropathologyA.• The correct answer is A, Wilson's disease

(hepatolenticular degeneration). • Wilson’s disease is caused by accumulation

BOARD FAVORITE!

• Wilson s disease is caused by accumulation of copper in the brain with decreased serum copper and ceruloplasmin and increased urine copper. Kayser-Fleischer rings form around the iris.

• Microscopy demonstrates Alzheimer type 2 t t hi h h l l i l astrocytes which have a large clear vesicular

nuclei and prominent nucleoli. Proliferation of large protoplasmic astrocytes such as Opalski cells may be seen.

• Research suggests that these reactive astrocytes may be involved in the process of

©™©™copper detoxification in WD.

Bertrand E, Lewandowska E, Szpak GM, Hoogenraad T, Blaauwgers HG, Czlonkowska A, Dymecki J. Neuropathological analysis of pathological forms of astroglia in Wilson's disease. Folia Neuropathol. 2001;39(2):73-9.

Degenerative disease, Wilson’s disease, Alzheimer type 2 astrocytes, Opalski cells,Alzheimer type 2 astrocytes, Opalski cells,

Copper

Page 37: Comprehensive Neurology Board Review Flash cards

NeuropathologyQ?• This muscle biopsy specimen is

consistent with:

NeuropathologyQ?

A. Congenital myopathyB. Myotonic dystrophyC. Steroid myopathyD. Mitochondrial myopathy

Note: Type 1 fibers labeled (1).1

©™©™

152

Page 38: Comprehensive Neurology Board Review Flash cards

NeuropathologyA NeuropathologyA.• The correct answer is C, steroid myopathy.• Steroid myopathy is characterized by

type 2 myofiber atrophy!

BOARD FAVORITE!

type 2 myofiber atrophy!

• Type 1 fiber atrophy- think muscle disease! -myotonic dystrophy and congenital myopathy

TYPE 2 ATROPHY

g y p y• Type 2 muscle atrophy- think neurogenic

or muscle disease! –denervation, disuse, myasthenia gravis, steroids and paraneoplastic syndrome (BOARD FAVORITE)

STEROID MYOPATHY

©™©™Griggs RC, Mendell JR, Miller RG: Congenital myopathies. In: Evaluation and Treatment of Myopathies. Philadelphia: FA Davis Co; 1995: 211-46.

Muscle, myopathy, steroid myopathy, type 2 fiber atrophy

Page 39: Comprehensive Neurology Board Review Flash cards

NeuropathologyQ?• This gross brain specimen is most

consistent with:

NeuropathologyQ?

A. AdrenoleukodystrophyB. Multiple sclerosisC. Mitochondrial myopathyD. Canavan’s disease

©™©™

154

Page 40: Comprehensive Neurology Board Review Flash cards

NeuropathologyA NeuropathologyA.• The correct answer is B, multiple sclerosis.• Multiple sclerosis (MS) is an idiopathic

(autoimmune?) inflammatory demyelinating

Periventricular location

BOARD FAVORITE!

(autoimmune?) inflammatory demyelinating disease of the CNS. Animal model experimental allergic encephalomyelitisdemonstrated by autoimmunity to myelin basic proteins. (BOARD FAVORITE)

• Episodes of de/remyelination result in a h i b d t l ith l ti chronic burned-out plaque with relative

preservation of axons and gliosis (recently axon transection has been reported in acute exacerbations).

• Incidence is higher in Caucasians. Female-to-male ratio is 2:1.

Chronic plaques

©™©™©™• Classic presentation -optic neuritis, transverse myelitis, internuclear ophthalmoplegia, paresthesias

Noseworthy JH, Lucchinetti C, Rodriguez M, Weinshenker BG: Multiple sclerosis. N Engl J Med 2000 Sep 28; 343(13): 938-52.

Demyelinating disease, Inflammatory disease, y g yMultiple Sclerosis, myelin basic protein

Page 41: Comprehensive Neurology Board Review Flash cards

NeuropathologyQ?• The gross spinal cord specimen (A)

is most consistent with:

NeuropathologyQ?

A. AdrenoleukodystrophyB. Multiple sclerosisC. Amyotrophic lateral sclerosisD. Neurofibromatosis

A B

©™©™

156

Page 42: Comprehensive Neurology Board Review Flash cards

NeuropathologyA NeuropathologyA.• The correct answer is C, amyotrophic

lateral sclerosis.• Amyotrophic lateral sclerosis

Ventral nerve root atrophy

BOARD FAVORITE!

Normal ventral roots

• Amyotrophic lateral sclerosisprogressive loss of upper and lower motor neurons.

• Pathology: progressive loss of anterior horn cells leads to atrophy of ventral nerve fibers and of skeletal muscles;

i l f i t

A B

progressive loss of primary motor neurons in motor cortex leads to degeneration of corticospinal/ corticobulbar tracts.

• Adrenoleukodystrophy (ALD)-leukodystrophy is an x-linked recessive disease due to deficiency in peroxisomal

©™©™lipid oxidation.

Gartner J, Braun A, Holzinger A , et al: Clinical and genetic aspects of X-linked adrenoleukodystrophy. Neuropediatrics 1998 Feb; 29(1): 3-13.

spinal cord, Degenerative disease, Amyotrophic lateral sclerosis, Upper motor neuron, Lowerlateral sclerosis, Upper motor neuron, Lower

motor neuron

Page 43: Comprehensive Neurology Board Review Flash cards

NeuropathologyQ?• This gross brain specimen shows

atrophy that is consistent with:

NeuropathologyQ?

A. Multiple sclerosisB. Pick’s diseaseC. Alzheimer’s diseaseD. Acute cerebral infarction

©™©™

157

Page 44: Comprehensive Neurology Board Review Flash cards

NeuropathologyA NeuropathologyA.• The correct answer is Alzheimer’s

disease.Al h i ’ di i th t

Frontal Atrophy

BOARD FAVORITE!

• Alzheimer’s disease is the most common dementing illness in adults, characterized by progressive dementia over several years. There is increased frequency with increasing age and in familial cases increasing age and in familial cases, usually earlier onset.

• Note the diffuse brain atrophy of Alzheimer's disease, unlike Pick’s disease which has mostly frontal lobe atrophy.

Temporal Atrophy

©™©™

p y• Risk is increased in Down’s

syndrome (BOARD FAVORITE). Kuljis RO: Modular corticocerebral pathology in Alzheimer's disease. In: Mangone CA, Allegri RF, Ariza, eds. Dementia: A Multidisciplinary Approach. 1997: 143-55.

Degenerative disease, Alzheimer’s disease, dementia, diffuse atrophydementia, diffuse atrophy

Page 45: Comprehensive Neurology Board Review Flash cards

NeuropathologyQ?• The pathology seen here is most likely due to:

A. Cerebellar astrocytoma

NeuropathologyQ?

B. AlcoholismC. TraumaD. Ruptured aneurysmE. Arteriovenous malformation

©™©™

158

Page 46: Comprehensive Neurology Board Review Flash cards

NeuropathologyA NeuropathologyA.• The correct answer is….• To obtain the answer to this question and to

BOARD FAVORITE!

view over 400 more comprehensive pathology questions please purchase the full product here!

©™©™Bolla L, Palmer RM: Paraneoplastic cerebellar degeneration. Case report and literature review. Arch Intern Med 1997 Jun 9; 157(11): 1258-62.

Degeneration, Cerebellum, Alcohol related degeneration, superior vermis of cerebellumdegeneration, superior vermis of cerebellum

Page 47: Comprehensive Neurology Board Review Flash cards

Neurobehavioral SectionNeurobehavioral Section

©™©™

Page 48: Comprehensive Neurology Board Review Flash cards

NeurobehavioralQ? Neurobehavioral

• Which of the following types of mental retardation is associated with a maternally-inherited deletion of segment 11-13 of chromosome 15?

Q?

A. Prader-Willi syndromeB. Rett’s syndromeC. Fragile X syndromeD. Angelman’s syndrome

©™©™

2

Page 49: Comprehensive Neurology Board Review Flash cards

NeurobehavioralA Neurobehavioral

• The correct answer is D, Angelman’s syndrome.• Angelman’s syndrome is classically associated with a maternally-inherited deletion of

segment 11 13 on chromosome 15 This s ndrome is characteri ed b mental retardation

A. BOARD FAVORITE!

segment 11-13 on chromosome 15. This syndrome is characterized by mental retardation, abnormal gait, speech impairment, seizures, and inappropriate happy behavior that is often described as “happy puppet syndrome.”

• Prader-Willi syndrome consists of the same deletion (segment 11-13 of chromosome 15) but is located on the paternal chromosome 15. (Remember: P-P, Paternal-Prader-Willi)

• Rett’s syndrome is a progressive neurodevelopmental disorder that affects females only The • Rett’s syndrome is a progressive neurodevelopmental disorder that affects females only. The cause of Rett’s syndrome is the MeCP2 gene, located on the long arm of the X chromosome.

• Fragile X syndrome is the most common form of inherited mental retardation. It is caused by a CGG repeat on the X chromosome, and effects include long face, enlarged ears, and macro-orchidism.

©™©™

Fauci A, Braunwald E, et al. Harrison’s Principles of Internal Medicine 17th Edition. New York, NY: McGraw-Hill, Medical Pub. Division 2005. p. 407-14.

Classification: Neurobehavioral & Psychiatry, mental retardation, geneticsy y, , g

Page 50: Comprehensive Neurology Board Review Flash cards

NeurobehavioralQ? Neurobehavioral

• Fragile X syndrome is the most common form of inherited mental retardation. Somatic abnormalities seen commonly in Fragile X patients include a long face, large ears, and macroorchidism Which of the following genetic abnormalities is the most likely cause of Fragile

Q?

macroorchidism. Which of the following genetic abnormalities is the most likely cause of Fragile X syndrome?

A. Absence of segment 11-13 on chromosome 5B. MeCP2 gene on the long arm of the X chromosomeC. CGG trinucleotide expansion of an area of the X chromosomeD I h i f f h 21D. Inheritance of an extra copy of chromosome 21

©™©™

3

Page 51: Comprehensive Neurology Board Review Flash cards

NeurobehavioralA Neurobehavioral

• The correct answer is C, CGG trinucleotide expansion of an area of the X chromosome.• Fragile X syndrome is the most common form of inherited mental retardation. It is caused

b a CGG repeat on the X chromosome and effects incl de long face enlarged ears and

A. BOARD FAVORITE!

by a CGG repeat on the X chromosome, and effects include long face, enlarged ears, and macro-orchidism. It affects males more than females.

• Angelman’s syndrome is classically associated with a maternally-inherited deletion of segment 11-13 on chromosome 15. This syndrome is characterized by mental retardation, abnormal gait, speech impairment, seizures, and inappropriate happy behavior that is often described as “happy puppet syndrome ” described as happy puppet syndrome.

• Prader-Willi syndrome consists of the same deletion (segment 11-13 of chromosome 15) but is located on the paternal chromosome 15.

• Rett’s syndrome is a progressive neurodevelopmental disorder that affects females only. The cause of Rett’s syndrome is the MeCP2 gene, located on the long arm of the X chromosome.

©™©™Sadock B, Sadock V. Kaplan & Sadock’s Synopsis of Psychiatry, 10th Edition. Philadelphia, PA: Lippincott Williams & Wilkins 2007. p. 1143.

Page 52: Comprehensive Neurology Board Review Flash cards

NeurobehavioralQ? Neurobehavioral

• A 6 year-old boy has been sent for evaluation by a psychiatrist for attention deficit hyperactivity disorder (ADHD) because he has not been completing his homework and is

t tl i t ti hi t h d l t Hi t h t th t h h

Q?

constantly interrupting his teacher and classmates. His teacher reports that he has trouble sitting still. Upon questioning his parents, they report that their son’s inattention and hyperactivity started over 6 months ago. With a diagnosis of attention deficit hyperactivity disorder in this child, which of the following would be the first-line treatment?

A. Individual psychotherapyB. Selective serotonin reuptake inhibitors & tricyclic antidepressantsC. Methylphenidate (Ritalin)D. Monoamine oxidase inhibitor (MAOI)

©™©™

4

Page 53: Comprehensive Neurology Board Review Flash cards

NeurobehavioralA Neurobehavioral

• The correct answer is C, methylphenidate (Ritalin).• Methylphenidate is considered the first line therapy for children diagnosed with

A. BOARD FAVORITE!

attention deficit hyperactivity disorder (ADHD). Significant improvement is seen in roughly 75% of children when taking methylphenidate. Individual psychotherapy with a focus on behavioral modification can be used as adjunctive therapy, but is not considered first-line. Selective serotonin reuptake inhibitor (SSRI) and tricyclic antidepressant (TCA) therapy can be used as adjunctive pharmacologic therapy for ADHD, but is not a first-line treatment. Monoamine oxidase inhibitors (MAOIs) are also antidepressants, but are not used in the treatment of attention deficit hyperactivity disorder. Of note, it is important to recognize that in order for a diagnosis of attention deficit hyperactivity disorder to be made, symptoms must be present for a minimum of 6 months, and the child must have the onset of symptoms before the age of 7.

©™©™Sadock B, Sadock V. Kaplan & Sadock’s Synopsis of Psychiatry, 10th Edition. Philadelphia, PA: Lippincott Williams & Wilkins 2007. p. 1211-14.

Page 54: Comprehensive Neurology Board Review Flash cards

NeurobehavioralQ? Neurobehavioral

• The treatment of attention deficit hyperactivity disorder (ADHD) classically involves the use of stimulants, such as methylphenidate (Ritalin). Which of the following are common d ff t f th l h id t ?

Q?

adverse effects of methylphenidate?A. Tachycardia, insomnia, and abdominal painB. Bradycardia and hyperphagiaC. Suicidal thoughts, gestures, or ideationD Increased intracranial pressure (ICP)D. Increased intracranial pressure (ICP)

©™©™

5

Page 55: Comprehensive Neurology Board Review Flash cards

NeurobehavioralA NeurobehavioralA.• The correct answer is A, tachycardia, insomnia, and abdominal pain.• Tachycardia, insomnia, and abdominal pain are all reported side effects of

BOARD FAVORITE!

methylphenidate. • Bradycardia is not seen. Methylphenidate is also reported to cause a decrease in

appetite, not hyperphagia. • Increased intracranial pressure is not a side effect of methylphenidate use.

©™©™Sadock B, Sadock V. Kaplan & Sadock’s Synopsis of Psychiatry, 10th Edition. Philadelphia, PA: Lippincott Williams & Wilkins 2007. p. 1211-12.

Page 56: Comprehensive Neurology Board Review Flash cards

NeurobehavioralQ? NeurobehavioralQ?• At which age are developmental milestones of group play, ability to ride a tricycle, and

the ability to copy a line or circle drawing present?A. 2 yearsB. 3 yearsC. 4 yearsD. 5 years

©™©™

7

Page 57: Comprehensive Neurology Board Review Flash cards

NeurobehavioralA NeurobehavioralA.• The correct answer is B, 3 years.• The above milestones are classically present at age 3.

BOARD FAVORITE!

• 4 year-olds, on the other hand, can participate in cooperative play and construct simple drawings (stick figure drawings, for example).

• Milestones like abstract reasoning and the formation of personality occur during adolescence (age 11 for girls and age 13 for boys).

©™©™Bhushan V, Le T, Chandwani R, Ozturk A. First Aid for the USMLE Step 1 2005: A Student to Student Guide. New York, NY: McGraw-Hill 2005. p. 120.

Page 58: Comprehensive Neurology Board Review Flash cards

NeurobehavioralQ? Neurobehavioral

• A 25 year-old medical student is arrested at school after stealing a laboratory scale. He has a history of multiple childhood arrests, both for abusing his neighbor’s dog and

tti fi t h b Hi h l d h hi t f di i li ti

Q?

setting fires to houses nearby. His school records show a history of disciplinary action for aggressiveness and fighting with classmates. During questioning, he shows no remorse for his actions. Prior to the age of 18, this individual would most probably have been diagnosed with which of the following?

A. Antisocial personality disorderB. Schizotypal personality disorderC. Conduct disorderD. Borderline personality disorderE. Histrionic personality disorder

©™©™

9

Page 59: Comprehensive Neurology Board Review Flash cards

NeurobehavioralA NeurobehavioralA.• The correct answer is C, conduct disorder. • Prior to age 18, individuals with the above behavior are classified as having conduct

BOARD FAVORITE!

disorder. Currently, as a 25 year-old medical student, he would be classified as having antisocial personality disorder.

• Borderline personality disorder is characterized by unstable moods and labile, unstable relationships.

• Histrionic personality disorder is characterized by excessive attention-seeking Histrionic personality disorder is characterized by excessive attention-seeking behavior and emotionality. These individuals are dramatic, sexually provocative, and unable to form long-lasting relationships.

• Schizotypal personality disorder is characterized by individuals with eccentricbehavior and peculiar thought patterns. They commonly exhibit odd beliefs or magical thinking

©™©™Sadock B, Sadock V. Kaplan & Sadock’s Synopsis of Psychiatry, 10th Edition. Philadelphia, PA: Lippincott Williams & Wilkins 2007. p. 1220-1224.

magical thinking.

Page 60: Comprehensive Neurology Board Review Flash cards

NeurobehavioralQ? Neurobehavioral

• Which of the following is a metabolic abnormality associated with anorexia nervosa?A. Hypochloremic hyperkalemic metabolic alkalosis

Q?

B. Hypochloremic hypokalemic metabolic alkalosisC. Hyperkalemic metabolic acidosisD. Hyponatremia

©™©™

11

Page 61: Comprehensive Neurology Board Review Flash cards

NeurobehavioralA NeurobehavioralA.• The correct answer is B, hypochloremic hypokalemic metabolic alkalosis.• Anorexia nervosa is an eating disorder in which body weight is 15% below normal.

BOARD FAVORITE!

Additionally, there is an intense fear of gaining weight, disturbed body image, and amenorrhea.

• The most common metabolic complication resulting from anorexia nervosa is hypochloremic hyperkalemic metabolic alkalosis. Lanugo, or fine body hair, is also commonly seen in the anorexic patient.y p

• Hypochloremic hypokalemic metabolic alkalosis is commonly seen in bulimia nervosa (secondary to emesis), not in anorexia.

• Hyponatremia and hyperkalemic acidosis are usually not seen in anorexia nervosa.

©™©™Sadock B, Sadock V. Kaplan & Sadock’s Synopsis of Psychiatry, 10th Edition. Philadelphia, PA: Lippincott Williams & Wilkins 2007. p. 731-2.

Page 62: Comprehensive Neurology Board Review Flash cards

NeurobehavioralQ? Neurobehavioral

• A 23 year-old female is brought in to your psychiatric clinic by her parents for an eating disorder evaluation. Her parents report that they rarely see her eating and are

d th t h i ff i f i Y f hi t d

Q?

concerned that she is suffering from anorexia nervosa. You perform a history and physical examination on the patient without her parents present, and find that she is mildly overweight. You notice that she has calloused knuckles on her right hand. When you ask her questions about her body image, she informs you that she is extremely distressed by her inability to achieve a lower body weight. What is the most lik l di i ?likely diagnosis?

A. Anorexia nervosaB. Bulimia nervosa, purging typeC. Bulimia nervosa, non-purging typeD This patient is not suffering from an eating disorder

©™©™D. This patient is not suffering from an eating disorder.

12

Page 63: Comprehensive Neurology Board Review Flash cards

NeurobehavioralA NeurobehavioralA.• The correct answer is B, bulimia nervosa, purging type.• This patient is suffering from bulimia nervosa, purging type.

C fi di i b li i ti t f th i t i l d h hl i h k l i

BOARD FAVORITE!

• Common findings in a bulimic patient of the purging type include hypochloremic hypokalemic metabolic alkalosis, dental erosion, and calloused knuckles (all as a result of self-induced vomiting).

• Interestingly, while anorexia nervosa patients have body weight 15% below-average, bulimia nervosa patients often have normal body weights or are slightly overweight.

• In addition, bulimia is described as ego-dystonic (distressing to the patient). These patients are aware , g y ( g p ) pthat they have a problem and are distressed by it, making them more likely to seek help than anorexic patients. Anorexia nervosa, on the other hand, is ego-syntonic (not distressing to the patient) and patients suffering from anorexia are less likely to seek help.

• The purging type of bulimia nervosa involves vomiting, the use of laxatives, or the use of diuretics as a means to lose weight. The non-purging type of bulimia involves excessive exercise or fasting. Note that while anorexia and bulimia can potentially involve excessive fasting the anorexic will have a low

©™©™

that while anorexia and bulimia can potentially involve excessive fasting, the anorexic will have a low body weight while the bulimic will have a normal body weight or be overweight.

Sadock B, Sadock V. Kaplan & Sadock’s Synopsis of Psychiatry, 10th Edition. Philadelphia, PA: Lippincott Williams & Wilkins 2007. p. 735-8.

Page 64: Comprehensive Neurology Board Review Flash cards

NeurobehavioralQ? Neurobehavioral

• A 23 year-old female gives birth to a healthy baby girl. However, on further inspection the newborn is noted to be deaf, has congenital cataracts, and has cardiac b liti ith ibl t l t d ti Th th t d h i

Q?

abnormalities with possible mental retardation. The mother reported having an unremarkable pregnancy except during the first trimester when she reported having a mild illness consisting of a maculopapular rash, arthritis, aching joints, and diffuse swollen lymph nodes lasting 2-3 days. These congenital anomalies could have best been prevented by which of the following?

A. Treatment of penicillin prior to pregnancyB. Receiving the measles, mumps, and rubella vaccine prior to pregnancyC. Avoiding uncooked pork and cat litter during the pregnancyD. Maintaining on highly active antiretroviral therapy (HAART) during the pregnancyE R i i th i ll t i i t

©™©™E. Receiving the varicella zoster vaccine prior to pregnancy

21

Page 65: Comprehensive Neurology Board Review Flash cards

NeurobehavioralA NeurobehavioralA.• The correct answer is B, receiving the measles, mumps and rubella vaccine prior to pregnancy.• This patient who was not immunized to rubella, contracted the disease during her pregnancy as noted

by her classic symptoms and classical congenital defects Congenital rubella syndrome (CRS) has the by her classic symptoms and classical congenital defects. Congenital rubella syndrome (CRS) has the highest transference rate during the first trimester.

• Choice A , refers to syphilis which usually manifests as a late abortion or congenital syphilis with eighth nerve deafness, saber shins, Hutchinson teeth, and a saddle nose

• Choice C, refers to Toxoplasmosis gondii a common protozoan parasite whose transmission also occurs during the first trimester. However, this parasite often manifests with fevers, seizures, chorioretinitis hepatosplenomegaly jaundice and hydrocephaly or microcephaly chorioretinitis, hepatosplenomegaly, jaundice, and hydrocephaly or microcephaly.

• Choice D, refers to HIV infected mothers. Transmission is believed to occur late in pregnancy or during delivery.

• Choice E, refers to the varicella zoster virus , which can also occur during the first trimester, but in this trimester the effects are often a spontaneous abortion and possible teratogenic effects but not the series of effects listed.

©™©™Charles R. B. Beckmann et al. Obstetrics and Gynecology Fifth Edition. Philadelphia, PA: Lippincott Williams & Wilkins 2006. p. 181- 184.

Page 66: Comprehensive Neurology Board Review Flash cards

NeurobehavioralQ? Neurobehavioral

• A 19 year old women presents to the psychiatrist with her mother. The patient reports having heard voices for the past month. On further questioning the psychiatrist discovers that for the past 2 months the patient has had grossly disorganized behavior and a delusion that someone

Q?

past 2 months the patient has had grossly disorganized behavior and a delusion that someone is following her. Her mother states that she was completely normal until two months ago. What is the most likely diagnosis at this time?

A. SchizophreniaB. Brief psychotic disorderC S hi ff ti di dC. Schizoaffective disorderD. SchizophreniformE. Schizotypical

©™©™

31

Page 67: Comprehensive Neurology Board Review Flash cards

NeurobehavioralA NeurobehavioralA.• The correct answer is D, schizophreniform.• While this patient displays the DSM IV criteria for schizophrenia, her symptoms have only

BOARD FAVORITE!

been present for the past 2 months. The DSM IV requires that symptoms be present for the past 6 months in order to diagnose schizophrenia (choice A). If the symptoms last under 1 month then the diagnosis of a brief psychotic disorder is made (choice B).

• Choice C, schizoaffective disorder is diagnosed when a patient meets both the DSM IV criteria for schizophrenia and a mood disorder. p

• Choice E, schizotypical is a personality disorder where a patient displays odd or magical beliefs, lack of friends, and the criteria for true psychosis is not met.

©™©™Benjamin James Sadock, M.D. and Virginia Alcott Sadock, M.D. Kaplan & Sadock’s Synopsis of Psychiatry. Philadelphia, PA: Lippincott Williams & Wilkins 2003. p. 471-511; 520-529; 806-807.

Page 68: Comprehensive Neurology Board Review Flash cards

NeurobehavioralQ? Neurobehavioral

• A nine year-old boy was adopted when he was one years old. His adopted papers showed that his mother was a recovering alcoholic and admitted to drinking heavily d i h Si h t t d h l t 6 h h b i i l

Q?

during her pregnancy. Since he started school at age 6, he has been in special education due to learning disabilities and an IQ test of 63. According to his IQ measurements, the boy would be classified as having:

A. Borderline intellectual functioningB. Mild mental retardationC. Moderate mental retardationD. Severe mental retardation

©™©™

47

Page 69: Comprehensive Neurology Board Review Flash cards

NeurobehavioralA NeurobehavioralA.• The correct answer is B, mild mental retardation.• In mild mental retardation, IQ ranges from 50/55 to approximately 70.

BOARD FAVORITE!

• In moderate mental retardation, IQ ranges from 35/40 to 50/55 • In severe mental retardation, IQ ranges from 20/25 to 35/40. • Borderline intellectual function is a term no longer used.

©™©™Shaywitz SE, Cohen DJ, Shaywitz BA. Behavior and learning difficulties in children of normal intelligence born to alcoholic mothers. J Pediatr. Jun 1980;96(6):978‐82

Page 70: Comprehensive Neurology Board Review Flash cards

NeurobehavioralQ?• A 45 year-old man is brought into the doctor’s office by his wife. She states that her

husband has been acting “weird” for the past few months. He does not eat food on the l ft id f hi l t d t h th l ft id f hi f d ill d th t h

NeurobehavioralQ?

left side of his plate, does not shave the left side of his face, and will deny that he even has a left arm. When the doctor asked the patient to draw a clock, he only drew a right sided clock. Shown here. What is the best diagnosis for this patient?

A. Gerstmann’s syndromeB. AgnosiagC. DementiaD. HemineglectE. Homonymous hemianopsia

©™©™

64

Page 71: Comprehensive Neurology Board Review Flash cards

NeurobehavioralA NeurobehavioralA.• The correct answer is D, hemineglect.• Neglect is lack of attention to one side, usually the left.

BOARD FAVORITE!

• Patients with severe neglect may fail to describe objects on the left, dress or shave their left side.

• They may even deny their left arm as theirs and claim that it is the examiner’s arm. • Patients with neglect do not have a hemianopia, or a primary motor or sensory deficit

for that side; rather they have damage to the non dominant (usually right) for that side; rather, they have damage to the non-dominant (usually right) hemisphere.

• Neglect is usually caused by lesions in the right frontal or parietal lobe

©™©™Guttmann-Steinmetz S, Crowell JA. Attachment and externalizing disorders: a developmental psychopathology perspective. J Am Acad Child Adolesc Psychiatry. Apr 2006;45(4):440-51.

Page 72: Comprehensive Neurology Board Review Flash cards

NeurobehavioralQ? Neurobehavioral

• A 26 year-old primigravid female has been admitted to the hospital today due to new-onset seizures that reportedly began five days ago. She has a history of rapid-cycling bi l di d Sh tl l i f d i t d ibl

Q?

bipolar disorder. She currently complains of depressive symptoms and possible suicidal ideation. The first year resident assigned to the case is interested in administering carbamazepine (Tegretol) as a mood stabilizer for her bipolar disorder. Which of the following adverse effects could carbamazepine potentially have on the developing fetus?

A. Growth deficiency and microcephalyB. Neonatal withdrawal syndromeC. Spina bifida and agranulocytosisD. Ebstein’s anomaly and other cardiac defectsE Fl b b d

©™©™E. Floppy baby syndrome

66

Page 73: Comprehensive Neurology Board Review Flash cards

NeurobehavioralA NeurobehavioralA.• The correct answer is….• To obtain the answer to this question and to view over 250 more comprehensive

BOARD FAVORITE!

neurobehavioral questions please purchase the full product here !

©™©™

Ebert M, Loosen P, Nurcombe B. Current Diagnosis & Treatment in Psychiatry. New York, NY: McGraw-Hill 2000. p. 320-1.

Classification: Neurobehavioral & Psychiatry, pharmacology, carbamazepine, t t i itteratogenicity

Page 74: Comprehensive Neurology Board Review Flash cards

NeuroradiologyQ?• A 47 year-old male presents with a complaint

of pain “shooting down” his right leg. The MRI is shown here Which of the following is

NeuroradiologyQ?

MRI is shown here. Which of the following is the most appropriate management for this patient?

A. Do nothing, MRI looks normalB. Laminectomy and discectomyC Laminectomy and instrumentation of C. Laminectomy and instrumentation of

L5-S1D. Tethered cord release

©™©™

2

Page 75: Comprehensive Neurology Board Review Flash cards

NeuroradiologyA NeuroradiologyA.• The correct answer is B, laminectomy and

discectomy.• Lumbar spine MRI shows a right far lateral

BOARD FAVORITE!

• Lumbar spine MRI shows a right far-lateral herniated disc (T2 axial view). Note that the disc bulge is not noted on the midline sagittal view.

• Far-lateral herniated discs commonly impinge on the exiting nerve root (same level) while

t l di ff t th t iti more central discs affect the transiting nerve root (level below).

• These patients generally do well with simple laminectomy and discectomy. There is no need to perform laminectomy and instrumentation of L5-S1. There is no evidence of a thickened

©™©™filum or low conus suggestive of a tethered cord.

H. Richard Winn, M.D. Youman’s Neurological Surgery 5th Edition. Philadelphia, PA: Elsevier 2004. p. 4514-7.

Classification: Neuroradiology, Herniated Disc, Laminectomy and Discectomy

Page 76: Comprehensive Neurology Board Review Flash cards

NeuroradiologyQ?• This MRI of the brain is most suggestive of:

A. Iron deposition

NeuroradiologyQ?

B. MacrocraniaC. HypoxemiaD. Generalized atrophyE. Butterfly glioma

©™©™

3

Page 77: Comprehensive Neurology Board Review Flash cards

NeuroradiologyA NeuroradiologyA.• The correct answer is A, iron deposition.• Hallervorden-Spatz syndrome - involves the

d iti f i i ll i th l b

BOARD FAVORITE!

deposition of iron especially in the globus pallidus and the retina.

• Dystonia, rigidity and neurobehavioral changes• MRI: Globus pallidus on T2-weighted images • "Eye-of-the-tiger" sign• Central region of hyperintensity

– Primary tissue insult – Produces edema

• Surrounding hypointensity Region high in iron

©™©™– Region high in iron – May be 2° process

H. Richard Winn, M.D. Youman’s Neurological Surgery 5th Edition. Philadelphia, PA: Elsevier 2004. p. 2720.

Classification: Neuroradiology, Hallervorden Spatz Syndrome Iron Hallervorden Spatz Syndrome, Iron Deposition

Page 78: Comprehensive Neurology Board Review Flash cards

NeuroradiologyQ?• Based on the AP and lateral cerebral

angiogram of the right external carotid artery shown here the most appropriate

NeuroradiologyQ?

artery shown here, the most appropriate statement to tell this patient is that:

A. The risk of bleeding is high due to the cortical venous drainage.

B. The risk of bleeding is low due to the cortical venous drainagecortical venous drainage.

C. If the flow has rapid drainage into a sinus the bleeding risk is increased.

D. This is a normal cerebral angiogram.

©™©™

4

Page 79: Comprehensive Neurology Board Review Flash cards

NeuroradiologyA NeuroradiologyA.• The correct answer is A. The risk of bleeding

is high due to the cortical venous drainage.• This AP and lateral angiogram shows a dural

BOARD FAVORITE!

• This AP and lateral angiogram shows a dural arteriovenous fistula with main feeders from the occipital and middle meningeal arteries. Multiple venous lakes and varices are identified including two large lesions (these suggest high pressure). Drainage of the dural AVF is via cortical veins primarily into the AVF is via cortical veins primarily into the sagittal and transverse sinuses.

• The risk of bleeding is high not low with cortical venous drainage.

• If the flow has rapid drainage into a sinus the bleeding risk decreases.

©™©™Borden JA, Wu JK, Shucart WA. A proposed classification for spinal and cranial dural arteriovenous fistulous malformations and implications for treatment. J Neurosurg. 1995 Feb;82(2):166-79.

Occipital artery

Classification: Neuroradiology, duralarteriovenous fistula cortical venous arteriovenous fistula, cortical venous drainage

Page 80: Comprehensive Neurology Board Review Flash cards

NeuroradiologyQ?• Locate the central sulcus:

A. A

NeuroradiologyQ?

B. BC. CD. DE. E AF. F B

CDE

F

©™©™

8

Page 81: Comprehensive Neurology Board Review Flash cards

NeuroradiologyA NeuroradiologyA.• The correct answer is C.• Sigmoid “Hook” or “Ducky Sign”• Hook like configuration (B) of the posterior

BOARD FAVORITE!

• Hook-like configuration (B) of the posterior surface of the precentral gyrus. The “hook” or “ducky’s breast” corresponds to the motor hand area. The “hook” is seen well on CT (89%) and MRI (98%).

• Pars bracket sign A• The paired pars marginalis (F) form a

“bracket” to each side of the interhemispheric fissure at or behind the central sulcus (96%).

• A = precentral sulcus• D = postcentral gyrus• E = postcentral sulcus

BCD

EF

©™©™• E = postcentral sulcus

Colen CB, Handbook of Neurosurgery and Neurology in Pediatrics, 2006

Classification: Neuroradiology, Neuroanatomy, Location of Central Sulcus

Page 82: Comprehensive Neurology Board Review Flash cards

NeuroradiologyQ?• A 2 day old asymptomatic neonate is

transferred to your institution with the f ll i i Whi h f th f ll i i

NeuroradiologyQ?

following image. Which of the following is CORRECT regarding this pathology?A. It rarely presents with congestive heart

failure.B. It never presents in older children. pC. The embryonic correlate to this

malformation is the median prosencephalic vein.

D. Treatment for this neonate should include immediate embolization

©™©™include immediate embolization.

11

Page 83: Comprehensive Neurology Board Review Flash cards

NeuroradiologyA NeuroradiologyA.• The correct answer is C, the embryonic correlate to this

malformation is the median prosencephalic vein.• Vein of Galen aneurysmal malformations (VGAM) probably

BOARD FAVORITE!

• Vein of Galen aneurysmal malformations (VGAM) probably represent an arteriovenous fistula (AVF) in the wall of a persistent embryonic vascular channel called the median prosencephalic vein.

• By week 10 of normal fetal development, the median prosencephalic vein regresses as the definitive internal cerebral

i A d l t i th i f G lveins appear. A caudal remnant remains as the vein of Galen.• Neonatal presentation with congestive heart failure is frequent

with these malformations.

Vein of Galen aneurysmal malformation

•Initial treatment of VGAM is conservative. Embolisation of a neonate is a high risk procedure and the child should be treated medically (for cardiac failure) until aged 5 or 6 months with regular outpatient assessment. Elective embolisation can be scheduled for this time with the aim of closing the AVS with cyanoacrylate glue. If the infant deteriorates ( i f il t th i i di f il t ) t t t i f d li

©™©™Lasjuanias P. Vascular Diseases in Neonates, Infants and Children. New York: Springer Verlag 1997. Horowitz MB, Jungreis CA, Quisling RG, Pollock I. vein of Galen aneurysms: a review and current perspective. AJNR 1994; 15:1486-1496. H. Richard Winn, M.D. Youman’s Neurological Surgery 5th Edition. Philadelphia, PA: Elsevier 2004. p. 3434.

(seizures, failure to thrive, worsening cardiac failure, etc) treatment is performed earlier.

Classification: Neuroradiology, Vascular Pathology Vein of Galen AneurysmalPathology, Vein of Galen AneurysmalMalformations

Page 84: Comprehensive Neurology Board Review Flash cards

NeuroradiologyQ? Neuroradiology

• A five-month-old male infant presents with an abnormal head shape. What is the most likely di i ?

Q?

diagnosis?A. Sagittal synostosisB. Bicoronal synostosisC. Left unicoronal synostosisD Right unicoronal synostosisD. Right unicoronal synostosisE. Metopic synostosis

©™©™

12

Page 85: Comprehensive Neurology Board Review Flash cards

NeuroradiologyA NeuroradiologyA.• The correct answer is C, left unicoronal synostosis.• Coronal synostosis can occur either on the right or left side

(unicoronal) or both (bicoronal ) It frequently occurs prenatally

Contralateral•Frontal bossing•Displaced fontanelle

Ipsilateral•Flattened forehead•Raised brow(unicoronal), or both (bicoronal.) It frequently occurs prenatally,

and appears to occur more commonly in males. Restriction of normal cranial growth at one suture between the frontal and parietal bones produces a characteristic flattening of the forehead on the fused side, and a bulging of the forehead on the non-fused side.Thi b l i b i i lt f t th f

p(Harlequin Eye)•Temporal bulging

• This bulging, or bossing, is a result of compensatory growth of the contralateral (opposite side) coronal suture. Secondary bulging of the temporal region on the ipsilateral (same) side can also occur. The placement of the eyes within the orbit is also typically affected. The superior margin of the orbit on the synostosed side is raised, a feature known as Harlequin eyed f it

©™©™deformity.

H. Richard Winn, M.D. Youman’s Neurological Surgery 5th Edition. Philadelphia, PA: Elsevier 2004. p. 3300-01.

Classification: Neuroradiology, Infantile Pathology, Unicoronal Synostosis

Page 86: Comprehensive Neurology Board Review Flash cards

NeuroradiologyQ?• This x-ray demonstrates which of the following?

A. McRae’s line

NeuroradiologyQ?

B. McGregor’s lineC. Rule of SpenceD. Chamberlain’s line

BA

B

©™©™Used with permission from Handbook of Neurosurgery andNeurology in Pediatrics; By Chaim B. Colen, MD, PhD.27

Page 87: Comprehensive Neurology Board Review Flash cards

NeuroradiologyA NeuroradiologyA.• The correct answer is C, rule of Spence.• McRae’s line - foramen magnum diameter: 35 mm+/- 4

BOARD FAVORITE!

• McGregor’s line - line from the hard palate to the caudal most part of the opisthion (odontoid should be < 4 mm above this line > 4 mm suggests basilar impression).

• Rule of Spence - if the sum of A + B ≥7mm, consider disruption of the transverse ligament (requires rigid immobilization).

• Chamberlain’s line diagonal line from the hard palate to the posterior foramen • Chamberlain s line - diagonal line from the hard palate to the posterior foramen magnum (odontoid should not extend 1/3 of its height above this line).

©™©™Colen CB, Handbook of Neurosurgery and Neurology in Pediatrics, 2006.

Classification: Neuroradiology, Rule of Spence, Clinical Definition

Page 88: Comprehensive Neurology Board Review Flash cards

NeuroradiologyQ?• What is the most likely

diagnosis depicted in this MRI and MRS?

NeuroradiologyQ?

and MRS?A. Low grade gliomaB. Pleiomorphic xantho-

astrocytomaC. High grade gliomaD N f h bD. None of the above

©™©™

34

Page 89: Comprehensive Neurology Board Review Flash cards

NeuroradiologyA NeuroradiologyA.• The correct answer is C, high grade glioma. • N-acetylaspartate (NAA) is predominantly located in neurons and is thus decreased in all

l th t th t b di l d l d ith li t ll Fi di

BOARD FAVORITE!

neoplasms that cause the neurons to be displaced or replaced with malignant cells. Findings of numerous studies have demonstrated decreased NAA values in glial neoplasms.

• Choline (Cho) peak contains contributions from glycerophosphocholine, phosphocholine, and phosphatidylcholine components that are thought to reflect cellular membrane density and turnover. As in any process that leads to hypercellularity and increased membrane proliferation the Cho value is consistently elevated in gliomas proliferation, the Cho value is consistently elevated in gliomas.

• Lactate (Lac) indicates that cellular respiration has shifted from the oxidative metabolism of carbohydrates to nonoxidative metabolism. Increased reliance on anaerobic glycolysis is found in highly malignant tumors.

©™©™Law M, Hamburger M, Johnson G, Inglese M, Londono A, Golfinos J, Zagzag D, Knopp EA. Differentiating surgical from non-surgical lesions using perfusion MR imaging and proton MR spectroscopic imaging. Technol Cancer Res Treat. 2004 Dec;3(6):557-65. Review.

Classification: Neuroradiology, Magnetic Resonance Spectroscopy High Grade Resonance Spectroscopy, High Grade Glioma

Page 90: Comprehensive Neurology Board Review Flash cards

NeuroradiologyQ?• A 64 year-old black male presented with left sided headache and slurring of speech.

Head CT is shown. Which of the following would likely be found on physical i ti ?

NeuroradiologyQ?

examination?A. Contralateral hemiparesisB. Ipsilateral hemiparesisC. Restricted left (lateral) gazeD Restricted right (medial) gazeD. Restricted right (medial) gazeE. Locked-in-syndromeF. Both A and C

©™©™

49

Page 91: Comprehensive Neurology Board Review Flash cards

NeuroradiologyA NeuroradiologyA.• The correct answers are A, (C and F).• This patient has Millard Gubler syndrome!• REMEMBER - 7th nerve (7 letters in Millard) and - 6th nerve (6 letters in Gubler)• PLUS the corticospinal tract.• Millard-Gubler syndrome is associated with abducens (CN6) and facial nerve

(CN7) paralysis, as well as contralateral hemiplegia of the extremities. It involves unilateral damage to the inferior pons commonly caused by pontine infarction or unilateral damage to the inferior pons, commonly caused by pontine infarction or hemorrhage, and leads to damage of the above structures. The muscles of the ipsilateral side of the face are paralyzed, diplopia, internal strabismus, and loss of extroversion are also typically present.

©™©™Onbas O, Kantarci M, Alper F, Karaca L, Okur A. Millard-Gubler syndrome: MR findings. Neuroradiology. 2005 Jan;47(1):35-7.

Classification: Neuroradiology, Millard Gubler Syndrome, Physical Exam

Page 92: Comprehensive Neurology Board Review Flash cards

NeuroradiologyQ?• A 57 year-old male underwent transnasal/

transfacial ethmoidectomy for metastatic ll i P t ti l h

NeuroradiologyQ?

squamous cell carcinoma. Post-operatively he developed clear rhinorrhea. Lumbar pucture was performed daily for 3 days draining 20 cc each time. On the third day he became confused and lethargic. On examination he was non-verbal and l li d t i f l ti l H d CT i h localized to painful stimulus. Head CT is shown. What is his most likely diagnosis?A. Recurrent tumorB. MeningitisC. Tension pneumocephalus

©™©™

C. Tension pneumocephalusD. Normal postoperative scan

50

Page 93: Comprehensive Neurology Board Review Flash cards

NeuroradiologyA NeuroradiologyA.• The correct answer is C, tension pneumocephalus.• This head CT shows the Mt Fuji sign - twin peaks with bifrontal pneumocephalus.

The Mount Fuji sign is a finding that can be observed on computed tomographic (CT) The Mount Fuji sign is a finding that can be observed on computed tomographic (CT) scans of the brain (1), in which bilateral subdural hypoattenuating collections cause compression and separation of the frontal lobes. The collapsed frontal lobes and the widening of the interhemispheric space between the tips of the frontal lobes have the appearance of the silhouette of Mount Fuji hence, the Mount Fuji sign.

• Given this patient’s decline in mental status he is likely developing tension pneumocephalus pneumocephalus.

©™©™H. Richard Winn, M.D. Youman’s Neurological Surgery 5th Edition. Philadelphia, PA: Elsevier 2004. p. 616.Ommaya AK: Cerebrospinal fluid fistula and pneumocephalus. In: Wilkins RH, Rengachary SS, eds. Neurosurgery. 2nd ed. New York: McGraw-Hill; 1996: 2773-82

Classification: Neuroradiology, Tension Pneumocephalus, CT scan

Page 94: Comprehensive Neurology Board Review Flash cards

NeuroradiologyQ?• A 4 year-old boy is brought to the

emergency room after a fall. Lateral k ll i h t th i ht

NeuroradiologyQ?

skull x-ray is shown to the right. Which of the following is the most likely diagnosis?

A. EpidermoidB. Eosinophilic granulomap gC. HemangioblastomaD. Normal skull x-ray

©™©™

66

Page 95: Comprehensive Neurology Board Review Flash cards

NeuroradiologyA NeuroradiologyA.• The correct answer is B, eosinophilic granuloma.• AP view of the skull reveals an approximate 3 cm. circular lytic lesion overlying the left

f t i t l i ith ll d li t d l ti i ti f hi ti t i X

BOARD FAVORITE!

frontoparietal region with well delineated non-sclerotic margins, suggestive of histiocytosis X (eosinophilic granuloma).

• Differential Diagnosis: Solitary lytic lesion in the skull includes metastatic lesion, myeloma, fibrous dysplasia, tuberculosis, trauma, histiocytosis X, osteomyelitis, leptomeningeal cyst, and epidermoid/dermoid.E i hili l l ti l i ith ll d li t d l ti i d • Eosinophilic granuloma - lytic lesion with well delineated non-sclerotic margins and beveled edges.

• Epidermoids - lytic lesion with sclerotic margins.

©™©™Yang JT, Chang CN, Lui TN, Ho YS. Eosinophilic granuloma of the skull--report of four cases. Changgeng Yi Xue Za Zhi. Dec 1993;16(4):257-62.

Classification: Neuroradiology, Pediatric Pathology, Eosinophilic Granuloma

Page 96: Comprehensive Neurology Board Review Flash cards

NeuroradiologyQ?• Treatment for this lesion in a patient with

neurofibromatosis type -1 should consist of:

NeuroradiologyQ?

A. Removing the optic nerve and attached globe.

B. Removing the optic nerve and both globes.

C. Radiotherapy with 25 GrayC. Radiotherapy with 25 GrayD. Radiotherapy with 80 Gray

©™©™

67

Page 97: Comprehensive Neurology Board Review Flash cards

NeuroradiologyA NeuroradiologyA.• The correct answer is A, removing the optic

nerve and attached globe.

BOARD FAVORITE!

• MRI demonstrates a high intensity lesion of the distal right optic nerve, suggestive of optic glioma, especially in a patient with neurofibromatosis type 1.

• Treatment of optic glioma distal to the p gchiasm involves removing the optic nerve and attached globe.

• Treatment of optic glioma involving the chiasm involves removing the optic nerve, globe and radiotherapy (less than 8 Gray)

©™©™

globe and radiotherapy (less than 8 Gray). Board Favorite!

Hollander MD, FitzPatrick M, O'Connor SG, et al: Optic gliomas. Radiol Clin North Am 1999 Jan; 37(1): 59-71.

Classification: Neuroradiology, NF-1 MRI, Treatment Modalities

Page 98: Comprehensive Neurology Board Review Flash cards

NeuroradiologyQ?• This tumor most likely has elevated serum

levels of which of following:

NeuroradiologyQ?

A. Alpha-fetoproteinB. Carcinoembryonic antigen (CEA)C. Cancer antigen-125 (CA-125)D. β-human chorionic gonadotropin (β-

HCG)HCG)E. Placental alkaline phosphatase (PLAP)

©™©™

69

Page 99: Comprehensive Neurology Board Review Flash cards

NeuroradiologyA NeuroradiologyA.• The correct answer is D, β-HCG.• T1W non-contrast MRI sagittal image shows a pineal

recess mass compressing the tectal plate and

BOARD FAVORITE!

Teilum Concept

recess mass compressing the tectal plate and causing triventricular hydrocephalus.

• INTRACRANIAL GERM CELL TUMORS• Germinoma is the most common.• Most common tumor of pineal recess 40%• Males• Respond well to chemo and radiotherapy• Cancer antigen-125 (CA-125)- ovarian cancer• Placental alkaline phosphatase (PLAP) - ovarian

cancer

©™©™Colen CB, Handbook of Neurosurgery and Neurology in Pediatrics, 2006 p. 41

Classification: Neuroradiology, Intracranial Germ Tumors, Serum levels

Page 100: Comprehensive Neurology Board Review Flash cards

NeuroradiologyQ?• A 20 year-old female presents

with 3 weeks of headaches, d iti MRI i

NeuroradiologyQ?

nausea, and vomiting. MRI is shown here. Which of the following is the most appropriate next step in management?A. Operative resectionB. No intervention neededC. Shunt placement or third

ventriculostomyD. RadiotherapyE A d C

©™©™E. A and C

70

Page 101: Comprehensive Neurology Board Review Flash cards

NeuroradiologyA NeuroradiologyA.• The correct answer is….• To obtain the answer to this question and to view over 250 more comprehensive

BOARD FAVORITE!

neuroradiology questions please purchase the full product here !

©™©™Barkovich AJ, Krischer J, Kun LE, et al: Brain stem gliomas: a classification system based on magnetic resonance imaging. Pediatr Neurosurg 1990-91;16(2): 73-83

Classification: Neuroradiology, TectalGlioma, Treatment Modalities

Page 102: Comprehensive Neurology Board Review Flash cards

NeurologyQ?• This patient most likely sustained

damage to which of the following t t ?

NeurologyQ?

structures?A. Bilateral hypoglossal nucleiB. Left hypoglossal nerveC. Left hypoglossal nucleusD Left upper motor neuron to the D. Left upper motor neuron to the

hypoglossal nucleusE. Right vagal nucleus

©™©™

2

Page 103: Comprehensive Neurology Board Review Flash cards

NeurologyA NeurologyA.• The correct answer is D, left upper motor neuron to the hypoglossal nucleus.• Damage to the left hypoglossal nucleus (or nerve) would cause tongue

BOARD FAVORITE!

deviation to the left (ipsilateral). However, as depicted in the photo, damage to the upper motor neurons (which cross), would cause tongue deviation to the right (contralateral).

©™©™Midstokke S, Hess SJ, Saini T, Edwards PC. Unilateral tongue atrophy. Gen Dent. 2006 Nov-Dec;54(6):425-7.

Classification: Neurology, upper motor neuron lesion, hypoglossal nucleus

Page 104: Comprehensive Neurology Board Review Flash cards

NeurologyQ?• The function of the superior olive is:

A. Sound localization

NeurologyQ?

B. ProprioceptionC. Vestibular localization and functionD. Visual gaze control

©™©™

6

Page 105: Comprehensive Neurology Board Review Flash cards

NeurologyA NeurologyA.• The correct answer A, sound localization.• The superior olivary nucleus (or superior

li ) i ll f b t

BOARD FAVORITE!

olive) is a small mass of gray substance situated on the dorsal surface of the lateral part of the trapezoid body.

• The superior olivary nucleus’ primary input is bilateral and from the bushy cells of the anterior ventral cochlear nuclei (AVCN)

Superior Olivary nucleus

Cochlear nucleus

Superior and Inferior colliculi

anterior ventral cochlear nuclei (AVCN). This input occurs primarily via the ventral acoustic stria. Its output is to the lateral lemnisci.

• The superior olivary nucleus is the first point where binaural input is combined.

Cochlear nucleus

Nucleus gracilis

Nucleus cuneatus

©™©™

where binaural input is combined.

Kandel ER, Schwartz JH, Jessell TM. Principles of Neural Science, 4th ed. McGraw-Hill, New York. 2000. p. 606.

Classification: Neurology, Superior Olive Nucleus, Function

Page 106: Comprehensive Neurology Board Review Flash cards

NeurologyQ?• The function of the superior olive is demonstrated by

which waveform:

NeurologyQ?

A. 1B. 2C. 3D. 4E 5

76

54

321

E. 5F. 6G. 7

©™©™

7

Page 107: Comprehensive Neurology Board Review Flash cards

NeurologyA NeurologyA.• The correct answer C, 3.• Brainstem auditory evoked responses test the

i t it f th dit th

BOARD FAVORITE!

integrity of the auditory pathway.• Wave 1 –auditory nerve• Wave 2 –Cochlear nuclei (pons)• Wave 3 –Superior olivary complex• Wave 4 –Lateral lemniscus

76

54

321

• Wave 5 –Inferior colliculus• Wave 6 –Medial geniculate nucleus• Wave 7 –Auditory radiations (cortex)• Damage to one of the structures will result in

an increased latency! (e g acoustic neuroma KNOW THESE WAVEFORMS WELL!

BOARD FAVORITE!

©™©™an increased latency! (e.g. acoustic neuroma will show increased latency of 1-3).

Kandel ER, Schwartz JH, Jessell TM. Principles of Neural Science, 4th ed. McGraw-Hill, New York. 2000. p. 606.

Classification: Neurology, Evoked Potential, Superior Olive

Page 108: Comprehensive Neurology Board Review Flash cards

NeurologyQ?• Match the following statements with the correct answer:

1. Lateral vestibulospinal tract

NeurologyQ?

2. Rubrospinal tract

A. Extensor toneB. Flexor toneC. BothD. Neither

©™©™

9

Page 109: Comprehensive Neurology Board Review Flash cards

NeurologyA NeurologyA.1. The correct answer is A, extensor tone. The lateral vestibulospinal tract regulates

extensor tone through the lateral vestibular nucleus.

BOARD FAVORITE!

• The vestibulospinal tract arises from the lateral vestibular nucleus (i.e. Deiter’s nucleus) and descends bilaterally in the anterior part of the lateral funiculus.

2. The correct answer is B, flexor tone. Flexor activity is regulated by the rubrospinal tract from the red nucleus.

• The rubrospinal tract arises from magnocellular neurons in the red nucleus and The rubrospinal tract arises from magnocellular neurons in the red nucleus and crosses at the ventral tegmental decussation. Stimulation of the red nucleus leads to excitation of contralateral flexor alpha motor neurons and inhibition of extensor alpha motor neurons.

©™©™Kandel ER, Schwartz JH, Jessell TM. Principles of Neural Science, 4th ed. McGraw-Hill, New York. 2000. p. 668.

Classification: Neurology, Motor Tracts, lateral vestibulospinal and rubrospinal tract

Page 110: Comprehensive Neurology Board Review Flash cards

NeurologyQ?• Which brainstem nuclei release serotonin?

A. Raphé nuclei

NeurologyQ?

B. Vestibular nucleiC. Hypoglossal nucleiD. Nucleus cuneatusE. Nucleus gracilis

©™©™

11

Page 111: Comprehensive Neurology Board Review Flash cards

NeurologyA NeurologyA.• The correct answer is A, raphé nuclei. The raphé nuclei are located in the brainstem.

Neurons in the dorsal raphé nucleus produce serotonin, they have long projections (green arrow) that carry the neurotransmitter to the orbital prefrontal cortex.

Dorsal raphé nuclei

CROSS SECTION

©™©™

Dorsal raphé nuclei

Classification: Neurology, Brainstem Nuclei, Serotonin Releasing Cells

Page 112: Comprehensive Neurology Board Review Flash cards

NeurologyQ?• Lack of sensory loss, presence of a “pinch sign”, and hand weakness is characteristic

of:

NeurologyQ?

A. Carpal tunnel syndromeB. Anterior interosseous syndromeC. Ulnar nerve entrapmentD. Posterior interosseous syndrome

©™©™

32

Page 113: Comprehensive Neurology Board Review Flash cards

NeurologyA NeurologyA.• The correct answer is B, anterior interosseous syndrome. The “pinch sign” often occurs when there

is a complete anterior interosseous nerve (AIN) lesion. Attempts to pinch the tips of the terminal phalanges of the index finger and thumb results in an extension of the distal phalanges Thus the

BOARD FAVORITE!

phalanges of the index finger and thumb results in an extension of the distal phalanges. Thus, the pulps rather than the tips of these two digits approximate.

Note the distinctive findings of a AIN syndrome on performing the pinch test. There is loss of function in the AIN-

NORMAL PINCH SIGN

©™©™

g y p g pinnervated flexor pollicis longus (FPL) and flexor digitorum profundus (FDP) muscles, leading to weakness in the distal phalanges of the thumb and forefinger. The left photograph shows the normal function when attempting this hand posture.

H. Richard Winn, M.D. Youman’s Neurological Surgery 5th Edition. Philadelphia, PA: Elsevier 2004. p. 3925..

Classification: Neurology, Diagnostic Tests, anterior interosseous syndrome

Page 114: Comprehensive Neurology Board Review Flash cards

NeurologyQ?• 30 year-old female presents with a history of shunted pseudotumor cerebri and small

ventricles, now complains of worsening headache. Fundoscopic exam is shown below. Whi h f th f ll i t t t i TRUE?

NeurologyQ?

Which of the following statements is TRUE?A. It is best treated with Diamox.B. She likely has shunt failure and stiff ventricles.C. She has venous thrombosis.D There is no cause for concern about this headacheD. There is no cause for concern about this headache.

©™©™

63

Page 115: Comprehensive Neurology Board Review Flash cards

NeurologyA NeurologyA.• The correct answer is B, she likely has shunt failure and stiff ventricles. A low index of

suspicion should be present when evaluating a patient with shunted pseudotumor cerebri (PTC) Approximately 40% of shunts placed in children will fail in the 1st year and almost all (PTC). Approximately 40% of shunts placed in children will fail in the 1st year and almost all children will require shunt revision at some point. In PTC, shunt malfunction may result in blindness.

• PTC is encountered most frequently in young, overweight women between the ages of 20 and 45. Headache, occurring in more than 90 percent of cases, is the most common presenting complaint Dizziness nausea and vomiting may also be encountered but there are typically complaint. Dizziness, nausea, and vomiting may also be encountered, but there are typically no alterations of consciousness or higher cognitive function.

• It is defined clinically by four criteria: (1) elevated intracranial pressure, as demonstrated by lumbar puncture; (2) normal cerebral anatomy, as demonstrated by neuroradiographic evaluation; (3) normal cerebrospinal fluid composition; and (4) signs and symptoms of increased intracranial pressure, including papilledema.

©™©™

p g p p

Martin TJ, Corbett JJ: Pseudotumor cerebri, in Youmans JR(ed): Neurological Surgery, ed 4. Philadelphia: WB Saunders,1996, Vol 4, pp 2980–2997.

Classification: Neurology, pseudotumorcerebri, shunt failure

Page 116: Comprehensive Neurology Board Review Flash cards

NeurologyQ? NeurologyQ?• Based on the radiological features seen in this MRI,

choose the most appropriate statement regarding this diticondition:

A. They have no intervening brain tissue between the vascular spaces on histopathology.

B. They do have intervening brain tissue between the vascular spaces on histopathology. p p gy

C. There is no genetic association.D. The HOX gene is highly associated.

©™©™

79

Page 117: Comprehensive Neurology Board Review Flash cards

NeurologyA NeurologyA.• The correct answer is A, they have no intervening brain tissue between the vascular spaces on

histopathology.• Cavernous malformations (i e angioma cavernoma or cavernous hemangioma) may be inherited or

BOARD FAVORITE!

• Cavernous malformations (i.e. angioma, cavernoma, or cavernous hemangioma) may be inherited or sporadic and consist of variable sized sinusoids or cavernous spaces between capillaries. Unlike arteriovascular malformations and capillary telangiectasias., they have no intervening brain tissuebetween the vascular spaces and have thus been described as “blood sponges”. MRI shows well-defined, usually rounded lesions with little or no mass effect and are without vasogenic edema (unless hemorrhage is present). There may be small areas of new or old hemorrhages shown as a rim of hemosiderosis around the cavernous angioma in the surrounding brain tissuehemosiderosis around the cavernous angioma in the surrounding brain tissue.

• Genetics: more common in Hispanics.• CCM1 (for cerebral cavernous malformation 1) -chromosome 7 at band 7q11.2-q21. It is also known as

KRIT1, for the protein created by the gene. 40% of familial cavernous angiomas. • CCM2 -band 7p15-p13, protein named malcavernin. 20% of familial cavernous angiomas. • CCM3 identified as linked to familial cavernous angioma is on chromosome 3 at band 3q

©™©™

CCM3 identified as linked to familial cavernous angioma is on chromosome 3 at band 3q.

Craig HD, Gunel M, Cepeda O, et al: Multilocus linkage identifies two new loci for a mendelian form of stroke, cerebral cavernous malformation, at 7p15-13 and 3q25.2-27. Hum Mol Genet 1998 Nov; 7(12): 1851-8.

Classification: Neurology, Cavernous Malformations, Histopathology

Page 118: Comprehensive Neurology Board Review Flash cards

NeurologyQ NeurologyQ.• This fundus is MOST likely seen in which of the

following patients?A. 25 year-old femaleB. 14 year-old diabetic maleC. 75 year-old diabetic maleD. 25 year-old obese maleE 25 ld b f lE. 25 year-old obese female

©™©™

80

Page 119: Comprehensive Neurology Board Review Flash cards

NeurologyA NeurologyA.• The correct answer is E, 25 year-old obese female.• This ocular fundoscopic image demonstrates classic

grade 2 papilledema most likely seen in a 25 year-old obese female with pseudotumor cerebri (idiopathic intracranial hypertension).

Grade II papilledema. The halo of edema now surrounds the optic disc.

©™©™Brazis PW, Lee AG: Elevated intracranial pressure and pseudotumor cerebri. Curr Opin Ophthalmol 1998 Dec; 9(6): 27-32.

Classification: Neurology, PseudotumorCerebri, Papilledema

Page 120: Comprehensive Neurology Board Review Flash cards

NeurologyQ?• In myopathic disorders the motor unit potentials on electromyography would most

likely show which of the following?

NeurologyQ?

A. Increased amplitudeB. Decreased durationC. MonophasicD. Few in numbers with decreased recruitmentE A d DE. A and D

©™©™

86

Page 121: Comprehensive Neurology Board Review Flash cards

NeurologyA NeurologyA.• The correct answer is….• To obtain the answer to this question and to view over 300 more comprehensive

BOARD FAVORITE!

neurology questions please purchase the full app here !

Polyphasic “giant wave” of reinnervation

Low amplitude wave of myopathy

©™©™Daube JR: The description of motor unit potentials in electromyography. Neurology 1978 Jul; 28(7): 623-5.

Classification: Neurology, Electromyography, Myopathic Disorders

Page 122: Comprehensive Neurology Board Review Flash cards

Neurocritical CareQ Neurocritical CareQ.• What is the hallmark indicator for disseminated intravascular coagulation (DIC)?

A. Decreased fibrin-split productsB. Increased factor XC. Low plateletsD. Decreased d-dimer

©™©™

5

Page 123: Comprehensive Neurology Board Review Flash cards

Neurocritical CareA Neurocritical CareA.• The correct answer is C, low platelets. Coagulation factors are decreased. Fibrin-split

products and d-dimers increase.Inciting factor: Ex. crush injury, endotoxin

Systemic activation of coagulation factors

Widespread intravascularfibrin deposition

Consumption of plateletsand clotting factors

Thrombosis & organ failure Thrombosis & organ failure

©™©™

DIC

Classification: Neurocritical Care, Coagulation Disorders, DICg ,

Page 124: Comprehensive Neurology Board Review Flash cards

Neurocritical CareQ?• A 36 year old male presents to the emergency room as a trauma code. During

resuscitation, clear fluid is noted dripping out of the left nare. An appropriate statement di th b d id l t t i th t it h

Neurocritical CareQ?

regarding the bedside glucose test is that it has:A. high sensitivity, low specificityB. high sensitivity, high specificityC. low sensitivity, low specificityD high sensitivity low specificityD. high sensitivity, low specificity

©™©™

22

Page 125: Comprehensive Neurology Board Review Flash cards

Neurocritical CareA Neurocritical CareA.• The correct answer is C, low sensitivity, low specificity. The bedside glucose test has low

sensitivity and low specificity. Therefore, it should be used only for trying to rule out a CSF leak rather than ruling it inleak rather than ruling it in.

• In the emergent setting, when evaluating a patient with rhinorrhea, if the rhinorrhoea contains glucose, the specificity of the test for CSF can be improved by excluding other factors that increase the glucose concentration of nasal discharge. If the nasal discharge is not blood stained, the blood glucose (measured at the same time as CSF glucose) is <6 mmol L–1 and there are no other symptoms of upper respiratory tract infection such as mmol.L 1, and there are no other symptoms of upper respiratory tract infection, such as sneezing, nasal blockage, cough, sore throat, sputum, or purulent nasal discharge, then this increases the likelihood that the discharge contains CSF. If rhinorrhoea does not contain glucose, then either it does not contain CSF or CSF glucose concentrations are below the limit of detection by the sticks (false negative measurement).

• Beta-2 transferrin test is both highly sensitive and specific but would require a few days to ©™©™

g y p q yprocess the Western blot at most institutions.

Chan DT, Poon WS, IP CP, Chiu PW, goh KY. How useful is glucose detection in diagnosing cerebrospinal fluid leak? The rational use of CT and Beta-2 transferrin assay in detection of cerebrospinal fluid fistula. Asian J Surg. 2004 Jan;27(1):39-42.

Classification: Neurocritical Care, CSF Leak, Bedside Glucose Test,

Page 126: Comprehensive Neurology Board Review Flash cards

Neurocritical CareQ?• Name three functions of platelets:

A. Attach, agglutinate, agranulate

Neurocritical CareQ?

B. Adhere, activate, avascularizeC. Adhere, aggregate, agranulateD. Apoptosis, aggregate, agranulateE. Acidification, activate, agranulate

©™©™

34

Page 127: Comprehensive Neurology Board Review Flash cards

Neurocritical CareA

V l I jAdherenceAdherence

Vascular phaseVascular phase Platelet phasePlatelet phase

Neurocritical CareA.• The correct answer is C, adhere, aggregate, and agranulate (degranulate) (Hint AAA).

VasospasmVasospasm

Vessel InjuryVessel Injury

AgranulationAgranulation

vWf

GPIb-IX-V complexGPIb receptor

Collagen

Platelet

Endothelial cell

2

AgranulationgGpIIb/IIIa

fibrinogen

GPIb-IX-V complex

PhospholipaseC activation

+

Release of platelet-activation factors

AdhAdherence1

+•Platelets can be activated by binding to collagen, which is mediated directly through glycoprotein VI (GPVI) or indirectly via von Willebrand factor (VWF) binding to GPIb-V–IX. Signal transduction from any of these receptors leads to phospholipase C recruitment, which mediates calcium mobilization, platelet shape change, degranulation, and activation of

©™©™AggregationAggregationAdherenceAdherence

3

Yee DL, Bergeron AL, Sun CW, Dong JF, Bray PF. Platelet hyperreactivity generalizes to multiple forms of stimulation.J Thromb Haemost. 2006 Sep;4(9):2043-50

GPIIb/IIIa to allow binding of fibrinogen and platelet-platelet interactions.

Classification: Neurocritical Care, Coagulation Cascade, Platelet Functiong ,

Page 128: Comprehensive Neurology Board Review Flash cards

Neurocritical CareQ?1. Choose the most common inherited cause of post-operative bleeding?

A. Bernard-Soulier disease

Neurocritical CareQ?

B. Von Willebrand diseaseC. Hemophilia AD. Hemophilia BE. Leiden factor disease

2. Inheritance of this disease is mostly:A. Autosomal dominantB. Autosomal recessive

©™©™

36

Page 129: Comprehensive Neurology Board Review Flash cards

Neurocritical CareA Neurocritical CareA.1. The correct answer is B, von Willebrand disease.2. The correct answer is A, autosomal dominant.• Von Willebrand disease is the most common coagulation disorder. Inheritance is

autosomal dominant transmission in 90% of cases. Children born with the disease have either low levels (quantitative defect) of von Willebrand factor (vWF), or defective form (qualitative defect) of the same protein.

• Bernard-Soulier: defective or decreased expression of the glycoprotein Ib/IX/V complex on th f f th l t l t Thi l i th t f Will b d f t ( WF) d the surface of the platelets. This complex is the receptor for von Willebrand factor (vWF), and the result of decreased expression is deficient binding of vWF to the platelet membrane at sites of vascular injury, resulting in defective platelet adhesion.

• Factor V Leiden is the most common hereditary hypercoagulability disorder. In this disorder the Leiden variant of factor V, cannot be inactivated by activated protein C.

©™©™Franchini M. Advances in the diagnosis and management of von Willebrand disease. Hematology. 2006 Aug;11(4):219-25.

Classification: Neurocritical Care, Coagulation Disorders, Von Willebrand DiDisease

Page 130: Comprehensive Neurology Board Review Flash cards

Neurocritical CareQ?• You are consulted on a 22 year-old female who is one

day postpartum and has been admitted to the intensive it ith id d t hi d

Neurocritical CareQ?

care unit with widespread petechiae, purpura, and mucocutaneous bleeding. Head CT is shown on the right. Which of the following is INCORRECT regarding this condition?

A. Blood clotting mechanisms are activated throughout the body.

B. Thrombocytopenia may occur.C. Thrombocytosis is commonD. Fibrin-split products are increased.

©™©™

38

Page 131: Comprehensive Neurology Board Review Flash cards

Neurocritical CareA Neurocritical CareA.• The correct answer is C, thrombocytosis is common• Thrombocytopenia occurs, not thrombocytosis.• The immediate postpartum period is a highly thrombogenic state. There are many risk factors

(amniotic fluid) that can cause a dysregulated hypercoagulable condition resulting in disseminated intravascular coagulation (DIC). DIC occurs when blood clotting mechanisms are activated throughout the body. Small blood clots form throughout the body, exhausting the supply of blood clotting factors which become rare at sites of real tissue injury. Simultaneously, clot dissolving mechanisms are also increased. Hence, this disorder can result in both excessive bleeding and , gclotting. DIC may be caused by infections (especially gram-negative bacteria) , severe trauma, cancer, blood transfusions, and obstetrical complications.

• Pathophysiology: two main mechanisms- generation of thrombin (microvascular thrombosis and organ ischemia) and generation of plasmin (characterized mainly by hemorrhagic symptoms).

• Fibrin-split products are increased in DIC.

©™©™Levi M, de Jonge E, van der Poll T, ten Cate H. Advances in the understanding of the pathogenetic pathways of disseminated intravascular coagulation result in more insight in the clinical picture and better management strategies. Semin Thromb Hemost. 2001 Dec;27(6):569-75. Review.

Classification: Neurocritical Care, Coagulation Disorders, Post Partumg ,

Page 132: Comprehensive Neurology Board Review Flash cards

Neurocritical CareQ? Neurocritical CareQ?• This ocular fundus is MOST likely seen in which

of the following patients?A. 25 year-old obese female punched in the

stomach with a GCS of 15B. 14 year-old male status post temporal

lobectomyC. 31 year-old male thrown from a 3rd story C. 31 year old male thrown from a 3rd story

window with a GCS of 5D. 56 year-old with a 1 cm cerebellar

hematoma with a GCS of 15

©™©™

46

Page 133: Comprehensive Neurology Board Review Flash cards

Neurocritical CareA Neurocritical CareA.• The correct answer is C, 31 year-old male thrown from a

3rd story window with a GCS of 5.Thi t f ill d i i th tti f hi h • This type of papilledema is seen in the setting of high intracranial pressure (ICP), such as the above patient who has suffered severe traumatic brain injury.

• Cerebellar hematoma acutely would cause high ICPs in the infratentorial compartment that potentially could result in acute obstructive hydrocephalus with papilledema but in acute obstructive hydrocephalus with papilledema, but this is less likely in a patient that has a 1 cm cerebellar hematoma and a GCS of 15.

Grade IV papilledema. There is severe swelling in addition to a circumferential halo. The edema covers major blood vessels as they leave the optic disk (grade III) and vessels on the disk (grade IV).A subretinal hemorrhage is present at 7 o'clock

©™©™Bhatt UK. Bilateral optic disc swelling; is a CT scan necessary? Emerg Med J. 2005 Nov;22(11):827-30.

A subretinal hemorrhage is present at 7 o clock. 

Classification: Neurocritical Care, Hematoma, Ocular Changes, g

Page 134: Comprehensive Neurology Board Review Flash cards

Neurocritical CareQ?• Which of the following is the preferred ventilation pattern in non-paralyzed patients

with central nervous system dysfunction?

Neurocritical CareQ?

A. Continuous mandatory ventilationB. Assist control ventilationC. Intermittent mandatory ventilationD. Synchronous intermittent mandatory ventilationE P t til tiE. Pressure support ventilation

©™©™

47

Page 135: Comprehensive Neurology Board Review Flash cards

Neurocritical CareA Neurocritical CareA.• The correct answer is D, synchronous intermittent mandatory ventilation.• Patients with CNS dysfunction tend to exhibit variations in tidal volume and respiratory

drive. • Synchronized intermittent mandatory ventilation (SIMV) delivers volume-cycled

breaths that coincide with spontaneous lung inflations. The patient can breathe unassisted and spontaneously between mechanical breaths. Also, a preselected respiratory rate can be used if the patient`s respiratory drive is insufficient to prevent p y p p y phypercarbia.

©™©™Stock MC, Perel A: Handbook of mechanical ventilatory support. Baltimore: Williams & Wilkins; 1997.

Classification: Neurocritical Care, CNS dysfunction, breathing patternsy , g p

Page 136: Comprehensive Neurology Board Review Flash cards

Neurocritical CareQ?• Which of the following characteristics are seen in patients with acute respiratory

distress syndrome (ARDS)?

Neurocritical CareQ?

A. Bilateral lung infiltratesB. Pulmonary capillary wedge pressure less than or equal to 18 mm Hg C. PAO2/FIO2 ratio of < 200 mm HgD. All of the above

©™©™

53

Page 137: Comprehensive Neurology Board Review Flash cards

Neurocritical CareA Neurocritical CareA.• The correct answer is D, all of the above.• Acute respiratory distress syndrome (ARDS) is characterized by acute onset, bilateral

BOARD FAVORITE!

infiltrates in chest x-ray, pulmonary capillary wedge pressure (PCWP) less than or equal to 18 mm Hg and a PAO2/FIO2 ratio < 200 mm Hg.

©™©™Kollef MH, Schuster DP: The acute respiratory distress syndrome. N Engl J Med 1995 Jan 5; 332(1): 27-37.

Classification: Neurocritical Care, Respiratory Distress Syndrome, Lab V lValues

Page 138: Comprehensive Neurology Board Review Flash cards

Neurocritical CareQ?• Which of the following findings will distinguish cardiogenic pulmonary edema from

acute respiratory distress syndrome (ARDS)?

Neurocritical CareQ?

A. Bilateral lung infiltrates on chest x-rayB. HypoxemiaC. Pulmonary capillary wedge pressure (PCWP) > 18 mm HgD. All of the above

©™©™

55

Page 139: Comprehensive Neurology Board Review Flash cards

Neurocritical CareA Neurocritical CareA.• The correct answer is C, Pulmonary capillary wedge pressure (PCWP) > 18 mm Hg.• Occasionally it is very difficult to distinguish between ARDS and cardiogenic

BOARD FAVORITE!

pulmonary edema. Both can present with hypoxia and bilateral lung infiltrate, but PCWP is > 18 mm Hg in cardiogenic pulmonary edema.

©™©™Ware LB, Matthay MA: The acute respiratory distress syndrome. N Engl J Med 2000 May 4; 342(18): 1334-49.

Classification: Neurocritical Care, Acute Respiratory Disress Syndrome, Pulmonary ChChanges

Page 140: Comprehensive Neurology Board Review Flash cards

Neurocritical CareQ?• What is most common finding on ECG in patients with pulmonary emboli?

A. Large Q wave in lead III

Neurocritical CareQ?

B. Inverted T wave in lead III C. Right axis deviationD. Wide S complex in lead IE. Right bundle branch blockF. Sinus tachycardia

©™©™

63

Page 141: Comprehensive Neurology Board Review Flash cards

Neurocritical CareA Neurocritical CareA.• The correct answer is F, sinus tachycardia.• All of the answers can be seen in case of pulmonary emboli. The classic findings of

BOARD FAVORITE!

“S1,Q3,T3” are not very sensitive in diagnosis of acute pulmonary embolism.

©™©™Feied CF: Pulmonary embolism. In: Rosen and Barkin, eds. Emergency Medicine Principles and Practice. Vol 3. 4th ed. 1998:chap 111.

Classification: Neurocritical Care, Pulmonary Emboli, ECG Changesy , g

Page 142: Comprehensive Neurology Board Review Flash cards

Neurocritical CareQ?• A 21 year-old patient with history of recurrent sinusitis developed an allergic reaction

after transfusion of a cross matched blood. What is the most likely cause?

Neurocritical CareQ?

A. Selective IgA deficiencyB. Lab errorC. Rh group incompatibilityD. Rapid transfusion of blood products

©™©™

66

Page 143: Comprehensive Neurology Board Review Flash cards

Neurocritical CareA Neurocritical CareA.• The correct answer is A, selective IgA deficiency.• Selective IgA deficiency syndrome is the most common primary immunodeficiency.

BOARD FAVORITE!

Usually the patient is asymptomatic, but a history of recurrent infections may be present; most commonly ear infections, sinusitis, and pneumonia. Allergies are another possible symptom including allergic reaction to blood products.

• If only fever develops within 1-6 hours of receiving cross matched blood, this is caused by antibodies in recipient blood that react to donor leukocytes. Leukocyte poor y p y y pred cells can be used to prevent this reaction.

©™©™Ballow M. Primary immunodeficiency disorders: antibody deficiency. J Allergy Clin Immunol. Apr 2002;109(4):581-91.Zhao SM. Clinical assessment of preventing febrile nonhemolytic transfusion reaction by leukocyte-depleted blood transfusion. 2002 Dec;10(6):568-70.

Classification: Neurocritical Care, Congenital Immune Disorders, IgA Def. S dSyndrome

Page 144: Comprehensive Neurology Board Review Flash cards

Neurocritical CareQ?• Which of the following is the appropriate immediate treatment in case of an intra-

operative venous air embolism?

Neurocritical CareQ?

A. Irrigation of the surgical fieldB. Lower the patient’s head with lateral left decubitus positionC. Aspiration of air from a multiforce central venous pressure catheterD. Manual occlusion of the jugular veinsE All f th bE. All of the above

©™©™

70

Page 145: Comprehensive Neurology Board Review Flash cards

Neurocritical CareA Neurocritical CareA.• The correct answer is E, all of the above.• Intraoperative venous air embolism (VAE) has a high incidence during procedures

BOARD FAVORITE!

performed in the sitting position. VAE is characterized by the development of bronchoconstriction, hypoxia, hypercarbia, hypotension, shock, cardiac arrhythmias, increased airway pressure, and decreased end tidal CO2.

• The most sensitive diagnostic modality for VAE is transesophageal echocardiography.g p y

©™©™Ballki M.Venous air embolism during awake craniotomy in a supine patient.Can J Anaesth. 2003 Oct;50(8):835-8.

Classification: Neurocritical Care, Intraoperative Venous Embolism, T t tTreatment

Page 146: Comprehensive Neurology Board Review Flash cards

Neurocritical CareQ?• Match the following arterial blood gas values with the appropriate acid/base disorder.

A. Metabolic acidosis

Neurocritical CareQ?

B. Metabolic alkalosisC. Respiratory acidosisD. Respiratory alkalosisE. Combined respiratory acidosis and metabolic acidosis

1. pH 7.26, PaCO2 53 mmHg, HCO3- 24 mEq/L

2. pH 7.49, PaCO2 46 mmHg, HCO3- 33 mEq/L

3. pH 7.30, PaCO2 33 mmHg, HCO3- 18 mEq/L

4. pH 7.27, PaCO2 41 mmHg, HCO3- 18 mEq/L

©™©™

75

Page 147: Comprehensive Neurology Board Review Flash cards

Neurocritical CareA Neurocritical CareA.• The correct answers are….• To obtain the answer to this question and to view over 250 more comprehensive

BOARD FAVORITE!

neurocritical care questions please purchase the full product here !

©™©™Narins RG, Emmett M: Simple and mixed acid-base disorders: a practical approach. Medicine (Baltimore) 1980; 59(3): 161-87.

Classification: Neurocritical Care, Acid Base Disorders, Lab Values,

Page 148: Comprehensive Neurology Board Review Flash cards

Neuropharmacology SectionNeuropharmacology Section

©™©™

Page 149: Comprehensive Neurology Board Review Flash cards

NeuropharmacologyQ?• A young man is brought to your clinic who has a history of seizures. He stopped taking

his medication for a while, but then decided to “double up” after having a seizure f th U h i l ti t i l d h

NeuropharmacologyQ?

recurrence a few months ago. Upon physical exam, you notice ataxia, slurred speech, and gingival proliferation.

• Which of the following seizure drugs is associated with the side effects mentioned above?

A. CarbamazepinepB. PhenytoinC. ValproateD. ZolpidemE. Phenobarbital

©™©™

1

Page 150: Comprehensive Neurology Board Review Flash cards

NeuropharmacologyA NeuropharmacologyA.• The correct answer is B, Phenytoin.

• Phenytoin acts by stabilizing the inactive state of calcium channels. • Although the other agents mentioned cause various cognitive shifts, only

phenytoin causes gingival proliferation.• Carbamazepine toxicity may cause hyponatremia, ataxia, nystagmus,

slurring of speech, dystonia, and varying degrees of CNS depression.• Valproate toxicity may cause coma confusion somnolence and cerebral • Valproate toxicity may cause coma, confusion, somnolence, and cerebral

edema. • Zolpidem toxicity may cause psychosis, distortion in visual perception, and lethargy.• Phenobarbital toxicity can cause fatal respiratory depression, nystagmus, and ataxia.

©™©™Amstrong L, Goldman M, Lacy C, Lance L. Drug Information Handbook. 13th Edition. 2005. pp 1195-1199

Classification: Neuropharmacology, antiseizure medication, phenytoin toxicity, p y y

Page 151: Comprehensive Neurology Board Review Flash cards

NeuropharmacologyQ?• A 61 year-old woman presents to your office complaining of shooting pains on the left

side of her face. The attacks of pain are so severe that she avoids brushing her teeth th l ft id d d ib lti l tt k th h t th d

NeuropharmacologyQ?

on the left side, and describes multiple attacks throughout the day. • Which of the following drugs would be considered first line treatment for her condition?

A. OxycodoneB. PhenytoinC EthosuximideC. EthosuximideD. CarbamazepineE. Phenobarbital

©™©™

2

Classification: Neuropharmacology, antiseizure medication, phenytoin toxicity, p y y

Page 152: Comprehensive Neurology Board Review Flash cards

NeuropharmacologyA NeuropharmacologyA.• The correct answer is D, Carbamazepine.• The patient is suffering from trigeminal neuralgia, which is best treated with

carbamazepine. • Carbamazepine is related to the structure of tricyclic antidepressants.• The method of action of this drug is related to the inhibition of voltage sensitive

calcium channels and the stabilization of sodium channels.• Trigeminal neuralgia is a debilitating disease characterized by neuropathy of the fifth• Trigeminal neuralgia is a debilitating disease characterized by neuropathy of the fifth

cranial nerve (trigeminal nerve). • Intense pain is experienced in the jaw, scalp, forehead, eyes, and nose.• Most patients who develop trigeminal neuralgia are over 40 years-old.• None of the other drugs listed are considered first line therapy for this condition.

©™©™

g py

A t L G ld M L C L L D I f ti H db k 13th Editi 2005 255 257Amstrong L, Goldman M, Lacy C, Lance L. Drug Information Handbook. 13th Edition. 2005. pp 255-257

Classification: Neuropharmacology, trigeminal neuralgia, carbamazepineg g , p

Page 153: Comprehensive Neurology Board Review Flash cards

NeuropharmacologyQ?• An eight year old boy is brought to your clinic. His mother mentions that her son often

drifts off in space, and has memory lapses during this period. EEG reveals a 3/second ik d tt

NeuropharmacologyQ?

spike and wave pattern.• You prescribe ethosuximide for this patient. Which of the following is the most likely

mechanism of action for this drug?A. Reduces current in T-type calcium channels on primary afferent neuronsB. Activates a hyperpolarizing potassium current and reduces voltage gated sodium B. Activates a hyperpolarizing potassium current and reduces voltage gated sodium

currentsC. Inhibits neuronal and glial uptake of GABAD. Enhances the inhibitory action of the GABA receptor.

©™©™

3

Page 154: Comprehensive Neurology Board Review Flash cards

NeuropharmacologyA NeuropharmacologyA.• The correct answer is A, reduces current in T-type calcium channels on primary

afferent neurons.• This patient is presenting with classic absence seizures.• Ethosuximide is a first line treatment for absence seizures. It reduces current in T-

type calcium channels on primary afferent neurons. Adverse effects include lethargy and GI problems.

• Topiramate activates a hyperpolarizing potassium current reduces voltage gated Topiramate activates a hyperpolarizing potassium current, reduces voltage gated sodium current, and increases postsynaptic GABA-A receptor currents.

• Tiagabine inhibits neuronal and glial uptake of GABA. It is used for treatment of partial seizures.

• Phenobarbital increases the inhibition of the GABA channel by increasing the length f ti th hl id h l i

©™©™of time the chloride channel remains open.

Amstrong L, Goldman M, Lacy C, Lance L. Drug Information Handbook. 13th Edition. 2005. pp 1488-1489

Classification: Neuropharmacology, antiseizure medication, ethosuximide MOA,

Page 155: Comprehensive Neurology Board Review Flash cards

NeuropharmacologyQ?• A young mother brings her one year old baby to the emergency room. The baby has

an elevated temperature of 102°F, has been vomiting, and just had a seizure. The b b h i d i i ti A i l t i f d Th CSF i l d

NeuropharmacologyQ?

baby has received no immunizations. A spinal tap is performed. The CSF is cloudy and the following lab values are obtained:

CSF glucose: 30CSF protein: 80 Opening pressure: 220

• Microbiology reports gram negative coccobacilli that are beta lactamase positive. Which is the most appropriate antibiotic regimen to begin in this patient?

A. VancomycinB. MoxifloxacinC Ceftriaxone

©™©™C. CeftriaxoneD. Penicillin G

4

Page 156: Comprehensive Neurology Board Review Flash cards

NeuropharmacologyA NeuropharmacologyA.• The correct answer is C, Ceftriaxone.

• This child has Haemophilus B (HiB) meningitis. • Although infection with HiB is declining due to the widespread use of the

vaccine, children who have not received the vaccine are still susceptible.• Ceftriaxone is a third generation cephalosporin with coverage for HiB. • Vancomycin is used against gram positive organisms. Haemophilus is a

ti igram negative organism.• Quinolones could be used to treat HiB, but they are not recommended for use in

children due inhibition of growth at the epiphyseal plate in some studies.• Penicillin G would not be used as lab reports indicated the bacteria, beta lactamase

resistant, was present. In addition, penicillin G does not have HiB coverage.©™©™

, p , p g

Amstrong L, Goldman M, Lacy C, Lance L. Drug Information Handbook. 13th Edition. 2005. pp288-289

Classification: Neuropharmacology, antimicrobials, third generation

h l icephalosporins

Page 157: Comprehensive Neurology Board Review Flash cards

NeuropharmacologyQ?• An eight year-old boy is taken by his father to the emergency room. An aseptic

meningitis is diagnosed by the ER physician. Physical exam shows i th th l t d th t d d ki h t th

NeuropharmacologyQ?

sores in the mouth located on the tongue and gums and a skin rash present on the palm of the patient’s hands and the soles of his feet. What treatment is most appropriate?

A. AmantadineB. AcycloviryC. TenofovirD. No treatment needed

©™©™

5

Page 158: Comprehensive Neurology Board Review Flash cards

NeuropharmacologyA• The correct answer is D, no treatment needed.• This patient has hand-foot-mouth disease, often caused by coxsackie virus A16,

NeuropharmacologyA.

which is a subgroup of the enteroviruses.• Currently there is no specific treatment available for any of the enteroviruses. • The clinical course of coxsackie A16 is relatively mild.• Amantadine is indicated for treatment of type A influenza. It is also used in

Parkinson’s diseaseParkinson s disease.• Acyclovir is the agent of choice in herpes simplex and herpes zoster infections.• Tenofovir is a nucleotide analogue reverse inhibitor used in HIV.

©™©™

Amstrong L, Goldman M, Lacy C, Lance L. Drug Information Handbook. 13th Edition. 2005. pp 43, 79-80, 1435

Classification: Neuropharmacology, Antiviral medications, Coxsackie A16,

Page 159: Comprehensive Neurology Board Review Flash cards

NeuropharmacologyQ?• A 35 year-old man has suffered from severe asthma attacks for most of his life. Which

of the following agents could cause a life threatening bronchospasm for this patient?

NeuropharmacologyQ?

A. PhenoxybenzamineB. TerazosinC. AcebutololD. Propanolol

©™©™

6

Page 160: Comprehensive Neurology Board Review Flash cards

NeuropharmacologyA• The correct answer is D, Propanolol.

• Propanolol is a non-selective beta blocker.

NeuropharmacologyA.

• Alpha mediated bronchoconstriction (α1) could thus take precedence over β2 relaxation.

• Phenoxybenzamine is a non selective α-blocker primarily used for treatment of pheochromocytoma. It causes no adverse effects in asthmatics.

• Terazosin is an α 1 adrenergic receptor antagonist for treatment of hypertension and • Terazosin is an α-1 adrenergic receptor antagonist for treatment of hypertension and BPH. It is not associated with any adverse effects in asthmatics.

• Acebutolol is a β -1 antagonist with sympathomimetic activity. It is especially indicated for patients who have asthma or diabetes.

©™©™Amstrong L, Goldman M, Lacy C, Lance L. Drug Information Handbook. 13th Edition. 2005. pp1267-1289

Classification: Neuropharmacology, autonomic agents, propanololg , p p

Page 161: Comprehensive Neurology Board Review Flash cards

NeuropharmacologyQ?• Which of the following drugs is given by intravenous infusion, has a very short half life,

and is used primarily during hypertensive crises?

NeuropharmacologyQ?

A. EsmololB. PrazosinC. CarvedilolD. Yohimbine

©™©™

7

Page 162: Comprehensive Neurology Board Review Flash cards

NeuropharmacologyA• The correct answer is A, Esmolol.

• Esmolol is a β-1 receptor antagonist for use in hypertensive crises.

NeuropharmacologyA.

• It can also be used during episodes of acute supraventricular tachycardia.• Prazosin is used for hypertension and benign prostatic hyperplasia. It is not indicated

for control of acute episodes of hypertension, however.• Carvedilol is an alpha and beta antagonist. At this time it is used to treat heart failure.

It is not given to control a hypertensive crisisIt is not given to control a hypertensive crisis.• Yohimbine is an a-2 agonist. At this time its clinical use is limited.

©™©™Amstrong L, Goldman M, Lacy C, Lance L. Drug Information Handbook. 13th Edition. 2005. pp 546-548

Classification: Neuropharmacology, autonomic agents, Esmololg ,

Page 163: Comprehensive Neurology Board Review Flash cards

NeuropharmacologyQ?• A 32 year-old soccer player injures his knee playing in the European World Cup. After

surgery to repair a torn ligament, he develops osteoarthritis two years later. He has a hi t f ll t lf d Whi h f th f ll i NSAIDS ld t b

NeuropharmacologyQ?

history of allergy to sulfa drugs. Which of the following NSAIDS would not be recommended in this patient?

A. AspirinB. IbuprofenC. SulindacC. SulindacD. Piroxicam

©™©™

8

Page 164: Comprehensive Neurology Board Review Flash cards

NeuropharmacologyA• The correct answer is C, Sulindac.• Sulindac is an NSAID that contains a sulfa moiety, and thus produces an allergic

NeuropharmacologyA.

reaction in those who are allergic to sulfa drugs.• None of the other choices contain a sulfa moiety.

©™©™

Amstrong L, Goldman M, Lacy C, Lance L. Drug Information Handbook. 13th Edition. 2005. pp 1413-1414

Classification: Neuropharmacology, Analgesics, Sulindacg ,

Page 165: Comprehensive Neurology Board Review Flash cards

NeuropharmacologyQ?• Which of the following drugs is used for treatment of myasthenia gravis?

A. Edrophonium

NeuropharmacologyQ?

B. PyridostigmineC. EchothiophateD. ParathionE. Donezipil

©™©™

10

Page 166: Comprehensive Neurology Board Review Flash cards

NeuropharmacologyA• The correct answer is B, Pyridostigmine.

• Pyridostigmine is a carbamate which acts as a pseudo-reversible inhibitor of

NeuropharmacologyA.

acetylcholinesterase. • It is rapidly hydrolyzed after carbamoylating the active site of the

acetylcholinesterase enzyme.• It is used extensively in the treatment of myasthenia gravis.

• Edrophonium is a reversible inhibitor of AchE Used for the diagnosis of myasthenia • Edrophonium is a reversible inhibitor of AchE. Used for the diagnosis of myasthenia gravis, but not for treatment due to an extremely short half life.

• Echothiophate is an irreversible inhibitor of AchE. Primarily utilized for glaucoma.• Parathion is an irreversible inhibitor of AChE. Primarily utilized for insect control.• Donezipil is a reversible inhibitor of AChE. Primarily utilized for treatment of

©™©™

p yAlzheimer’s disease.

Amstrong L, Goldman M, Lacy C, Lance L. Drug Information Handbook. 13th Edition. 2005. pp 1281-1282

Classification: Neuropharmacology, anticholinesterase drugs, pyridostigmineg , py g

Page 167: Comprehensive Neurology Board Review Flash cards

NeuropharmacologyQ?• Which variables alter the mechanism of intrathecal drug distribution?

A. Age

NeuropharmacologyQ?

B. WeightC. VicosityD. Patient positionE. All of the above.

©™©™

20

Page 168: Comprehensive Neurology Board Review Flash cards

NeuropharmacologyA NeuropharmacologyA.• The correct answer is E, all of the above.• Factors that influence the distribution of intrathecally administered medication

include:1. Characteristics of the injected solution: baricity, volume/dose/concentration,

temperature of injectate, viscosity, additives2. Clinical technique: patient position, level of injection, needle type/alignment,

intrathecal catheters fluid currents epidural injectionintrathecal catheters, fluid currents, epidural injection3. Patient characteristics: age, height, weight, sex, intra-abdominal pressure, spinal

anatomy, lumbosacral CSF volume, pregnancy

©™©™Greene NM. Distribution of local anesthetic solutions within the subarachnoid space. Anesth Analg 1985; 64: 715–30.

Classification: Neurosurgery, Intrathecal Drugs, Variables Affecting Mechanism of

Distribution

Page 169: Comprehensive Neurology Board Review Flash cards

NeuropharmacologyQ?• A 76 year-old male presents to your clinic in consultation with results of a recent

angiogram that suggest carotid stenosis 45% by NASCET criteria. He denies current i l h h d h k b t th t 2 k h did h “l

NeuropharmacologyQ?

visual changes, headaches or weakness, but says that 2 weeks ago he did have “loss of sight” in the left eye for one day . You would recommend:

A. Medical therapy (ie, antiplatelet agents and cardiovascular risk factor control/prevention).

B. Recommend carotid endarterectomy after consider patient's risk factor profile y p pand comorbidities.

C. Recommend endarterectomy without considering patient's risk factor profile and comorbidities.

D. Carotid endarterectomy plus antiplatelet therapy and cardiovascular risk factor control/prevention

©™©™control/prevention.

21

Page 170: Comprehensive Neurology Board Review Flash cards

NeuropharmacologyA NeuropharmacologyA.• The correct answer is A, medical therapy.• NASCET criteria for carotid stenosis:

BOARD FAVORITE!

• Symptomatic (TIA or minor stroke) • Lower-grade carotid stenosis (<50% by NASCET criteria) -medical therapy (ie, antiplatelet

agents and cardiovascular risk factor control/prevention)• Moderate-grade symptomatic carotid stenosis (50% to 69% by NASCET criteria) walk a finer

line. Consider patient's risk factor profile and comorbidities.• High-grade symptomatic carotid stenosis (70% to 99% by NASCET criteria) -CEA plus

antiplatelet therapy and cardiovascular risk factor control/prevention is the treatment of choice.• Asymptomatic• >60% carotid stenosis (NASCET criteria) walk a fine line. If surgical complications exceed 3%,

CEA would be harmful to the patient. ©™©™

p

Ferguson GG, Eliasziw M, Barr HWK, Clagett GP, et al. The North American Symptomatic Carotid Endarterectomy Trial. Stroke. 1999;30:1751-8.

Classification: Neurosurgery NASCET Classification: Neurosurgery, NASCET, Carotid Stenosis

Page 171: Comprehensive Neurology Board Review Flash cards

NeuropharmacologyQ? NeuropharmacologyQ?• A 75-year old man who had just suffered

an acute myocardial infarction one week previously now presents to clinic with tachycardia. His EKG is shown here. An appropriate treatment regimen would include which of the following drugs?

A. Magnesiumag es uB. ProcainamideC. AmiodaroneD. VerapamilE. Lidocaine

©™©™

31

Page 172: Comprehensive Neurology Board Review Flash cards

NeuropharmacologyA NeuropharmacologyA.• The correct answer is….• To obtain the answer to this question and to view over 250 more comprehensive

neuropharmacology questions please purchase the full product here !

©™©™Stoelting R, Hillier s. Pharmacology and Physiology in Anesthetic Practice 4th Edition. Philadelphia, PA: Lipincott 2006. p. 620

Category: Pharmacology, torsades de pointe, magnesium 

Page 173: Comprehensive Neurology Board Review Flash cards

NeurobiologyQ?• Match the following statements

with the correct answer:

NeurobiologyQ?

A A t l h li l1. Neuromuscular junction2. Botox3. Renshaw cell4. Blocks Renshaw cell

neurotransmitter

A. Acetylcholine releaseB. Acetylcholine receptorC. 5 subunits: 2α subunits, a β, a δ and either

a γ or an ε.D Strychnineneurotransmitter

5. CurareD. StrychnineE. Glycine

©™

4

Page 174: Comprehensive Neurology Board Review Flash cards

NeurobiologyA NeurobiologyA.• The corrects answers are the following: 1 - C, 2 - A, 3 - E, 4 - D, 5 - B. • Neuromuscular junction- the neuromuscular junction contains the nicotinic receptor

BOARD FAVORITE!

(an acetylcholine receptor subtype (nAChR)) There are 2 types of nAChRs; autonomic and neuromuscular. The autonomic nicotinic receptor has 5 subunits: 3α and 2β. The neuromuscular nAChR also consists of 5 subunits:2α subunits, a β, a δ and either a γ or an ε subunit.

• Botox –botulism toxin decreases Ach release (at neuromuscular junction and ( jautonomic nerve terminals: both sympathetic & parasympathetic) by binding to synaptotagmin, while curare (δ-tubocurarine) blocks the Ach receptor. Botox has actually become useful for cosmetic purposes and dystonias.

• The Renshaw cell neurotransmitter is glycine; an aminoacid and whose channel is blocked by strychnine

©™

blocked by strychnine.Dressler D, AdibSaberi F. New formulation of Botox: complete antibody-induced treatment failure in cervical dystonia. J NeurolNeurosurg Psychiatry. 2007 Jan;78(1):108-9Adams, M.E. and Olivera, B.M. Neurotoxins: overview of an emerging research technology. Trends Neurosci. 17:151-155. (1994).

Classification: Neurobiology, Neuromuscular junction, Receptor actions

Page 175: Comprehensive Neurology Board Review Flash cards

NeurobiologyQ?• Which of the following is the principle visceral nucleus of the brainstem?

A. Nucleus ambiguus

NeurobiologyQ?

B. Raphe nucleusC. Nucleus solitariusD. Locus ceruleusE. Trigeminal nucleus

©™

5

Page 176: Comprehensive Neurology Board Review Flash cards

NeurobiologyA NeurobiologyA.• The correct answer is C, nucleus solitarius.• Nucleus solitarius receives afferent visceral information from the heart, lungs, GI

BOARD FAVORITE!

tract, as well as for taste. The solitary nucleus and tract are neural structures in the brainstem that carry and receive visceral sensation and taste from the facial (VII), glossopharyngeal (IX), vagus (X) cranial nerves, as well as the cranial part of the accessory nerve (XI).

• Nucleus ambiguous gives rise to the efferent motor fibers of the vagus nerve (CN X) g g g ( )terminating in the laryngeal and pharyngeal muscles, efferent motor fibers of the glossopharyngeal nerve (CN IX) terminating in the stylopharyngeus and contains the preganglionic parasympathetic neurons for cardiac regulation.

©™Andresen MC, Doyle MW, Bailey TW, Jin YH. Differentiation of autonomic reflex control begins with cellular mechanisms at the first synapse within the nucleus tractus solitarius. Braz J Med Biol Res. 2004 Apr;37(4):549-58.

Classification: Neurobiology, Brainstem, Nucleus Solitarius

Page 177: Comprehensive Neurology Board Review Flash cards

NeurobiologyQ?• Choose the MOST accurate statement regarding the physiology of an action potential:

A. Voltage-gated potassium channels (also called delayed rectifier potassium

NeurobiologyQ?

channels) have a delayed response, such that potassium continues to flow out of the cell and initiate depolarization.

B. Hyperpolarization is caused by K+ influx into a cell.C. Closing of voltage-gated potassium channels is both voltage- and time-

dependent.dependent.D. Depolarization is caused by Na+ efflux from a cell.

©™

35

Page 178: Comprehensive Neurology Board Review Flash cards

NeurobiologyA NeurobiologyA.• The correct answer is C, closing of voltage-gated potassium channels is both voltage- and time-

dependent. As potassium exits the cell, the resulting membrane repolarization initiates the closing of voltage-gated potassium channels. These channels do not close immediately in

BOARD FAVORITE!

Peak

ne v

olta

ge (m

V)

40

0

• These channels do not close immediately in response to a change in membrane potential. Rather, voltage-gated potassium channels (also called delayed rectifier potassium channels) have a delayed response, such that potassium continues to flow out of the cell even after the membrane has fully repolarized. Thus the membrane potential

Hyperpolarization

Resting potential

0 1 2 3 4 5

Threshold

Stimulus

Mem

bran

-55

-70

Na+ influx

K+ effluxrepolarized. Thus the membrane potential dips below the normal resting membrane potential of the cell for a brief moment; this dip of hyperpolarization is known as the undershoot.

• Hyperpolarization is caused by K+ effluxfrom a cell.

• Depolarization is caused by Na+ influx into a ©™

Time (ms)Depolarization is caused by Na influx into a cell.

Kandel ER, Schwartz JH, Jessell TM. Principles of Neural Science, 4th ed. McGraw-Hill, New York. 2000. p. 128-33.Robbins and Cotran. Pathologic Basis of Disease, 6th Edition. W.B. Saunders Company. 1999. p.55-56.

Classification: Neurobiology, Neurophysiology, Action Potential

Page 179: Comprehensive Neurology Board Review Flash cards

NeurobiologyQ?• Neurotransmitter that is found in a postganglionic sympathetic nerve terminal:

A. GABA

NeurobiologyQ?

B. SerotoninC. AcetylcholineD. NorepinephrineE. C + D

©™

36

Page 180: Comprehensive Neurology Board Review Flash cards

NeurobiologyA NeurobiologyA.• The correct answer is E, acetylcholine (Ach) and norepinephrine (NE).• Both Ach and NE are found in postganglionic sympathetic fibers.

BOARD FAVORITE!

• Ach- sweat, piloerectors, and vasculature, mediated by muscarinic receptors (inhibited by pertussis toxin).

• REMEMBER: sweating of the palms of the hands is parasympathetic NOT sympathetic! (BOARD FAVORITE!)

• NE all others• NE- all others• Peripheral sympathetic pathway –TRAVELS WITH SKELETAL NERVES (sweat,

piloerectors, and vasculature) (BOARD FAVORITE!)• Head and Neck sympathetic pathway -TRAVELS WITH BLOOD VESSELS

(BOARD FAVORITE!)©™• All preganglionic fibers (both sympathetic and parasympathetic) use Ach.

Kandel ER, Schwartz JH, Jessell TM. Principles of Neural Science, 4th ed. McGraw-Hill, New York. 2000. p. 1051-52.

Cl ifi ti N bi l Classification: Neurobiology, Neurophysiology, Sympathetic Nerve Terminal

Page 181: Comprehensive Neurology Board Review Flash cards

NeurobiologyQ?• Which of the following statements is true regarding the intracellular second

messenger cyclic AMP.

NeurobiologyQ?

A. It is involved with photoreceptionB. It is hydrolyzed by phospholipaseC. It synergistically activates protein kinase CD. It is increased with D1 receptor stimulation

©™

37

Page 182: Comprehensive Neurology Board Review Flash cards

NeurobiologyA NeurobiologyA.• The correct answer is D, it is increased with D1 receptor stimulation.• Hormones that utilize this system include: epinephrine and norepinephrine, glucagon

and TSH

BOARD FAVORITE!

and TSH.• In a cAMP dependent pathway, the activated Gs alpha subunit binds to and activates

an enzyme called adenylyl cyclase which in turn catalyzes the conversion of ATP into cyclic adenosine monophosphate (cAMP). The intracellular second messenger cAMP activates protein kinase A (PKA).

• cAMP is decomposed to AMP by the action of the enzyme phosphodiesterase. This cAMP phosphodiesterase has an activity opposite to kinase therefore it cAMP phosphodiesterase - has an activity opposite to kinase, therefore it dephosphorylates cAMP into AMP, reducing the cAMP levels.

• Deregulation of cAMP pathways (hyperactive) and an aberrant activation of cAMP-controlled genes is linked to the growth of some cancers

• Activation of conventional protein kinase C - requires DAG, Ca2+, and phospholipid.

©™Kandel ER, Schwartz JH, Jessell TM. Principles of Neural Science, 4th ed. McGraw-Hill, New York. 2000. p. 230-36.

Cl ifi ti N bi l Classification: Neurobiology, Neurophysiology, Second Messenger Systems

Page 183: Comprehensive Neurology Board Review Flash cards

NeurobiologyQ?• Match the receptors with their action:

1. Mu1

NeurobiologyQ?

A. Hallucinations2. Mu23. Kappa4. Delta5. Sigma

B. Opiate analgesic euphoriaC. Miosis D. Reinforcing behaviorE. Respiratory depression

©™

38

Page 184: Comprehensive Neurology Board Review Flash cards

NeurobiologyA NeurobiologyA.• The correct answers are:

1. B, Mu1- Opiate analgesic euphoria

BOARD FAVORITE!

2. E, Mu2- Respiratory depression3. C, Kappa- Miosis4. D, Delta- Reinforcing behavior5. A, Sigma- Hallucinations

©™Kandel ER, Schwartz JH, Jessell TM. Principles of Neural Science, 4th ed. McGraw-Hill, New York. 2000. p. 483-6.Robbins and Cotran. Pathologic Basis of Disease, 6th Edition. W.B. Saunders Company. 1999. p.55-56.

Classification: Neurobiology, Pharmacology, Receptor Actions

Page 185: Comprehensive Neurology Board Review Flash cards

NeurobiologyQ?• Hexamethonium blocks which receptor?

A. Muscarinic

NeurobiologyQ?

B. NicotinicC. Glycine D. Glutamate

©™

39

Page 186: Comprehensive Neurology Board Review Flash cards

NeurobiologyA NeurobiologyA.• The correct answer is B, nicotinic receptor.• Hexamethonium is a nicotinic Ach receptor antagonist, acts by preventing

acetylcholine from binding to the cholinergic receptor

BOARD FAVORITE!

acetylcholine from binding to the cholinergic receptor.• It has no effect on the muscarinic acetylcholine receptors, or mAChRs.

©™Kandel ER, Schwartz JH, Jessell TM. Principles of Neural Science, 4th ed. McGraw-Hill, New York. 2000. p. 971.

Classification: Neurobiology, Pharmacology, Receptor Actions

Page 187: Comprehensive Neurology Board Review Flash cards

NeurobiologyQ?• A 37 year-old male presented with generalized seizure and expressive dysphasia.

MRI is shown. Reoccurrence of this tumor as a high-grade is LEAST likely due to hi h ti b lit ?

NeurobiologyQ?

which genetic abnormality?A. P53 tumor suppressorB. Retinoblastoma geneC. Amplification of platelet

derived growth factorderived growth factorD. Amplification of epidermal

growth factor receptor

©™

41

Page 188: Comprehensive Neurology Board Review Flash cards

NeurobiologyA NeurobiologyA.• The correct answer is D, amplification of epidermal growth factor receptor.• This MRI demonstrates a low-grade glioma. Recurrent progressive low-grade glioma is

i t d ith t ti i th t 53 ith lifi ti f CDK4

BOARD FAVORITE!

associated with mutations in the tumor suppressor gene p53 with amplification of CDK4 or loss of Rb and amplification or overexpression of PDGF.

• There are 2 subtypes of glioblastoma– Older patients with de novo tumors

• Deletions in cell cycle related INK4a-ARF genes p16INK4A and p19/p14ARF• EGFR amplifications

– Younger patients with progressive low grade glioma• Mutations in the tumor suppressor gene p53 with amplification of CDK4 or loss of

Rb• Amplification or overexpression of PDGF

©™

Amplification or overexpression of PDGF

Berger MS, Pardos M,Textbook of Neuro-Oncology. Saunders, Philadelphia. 2005. p. 608-9.

Classification: Neurobiology, Genetics, Tumor Growth Factors

Page 189: Comprehensive Neurology Board Review Flash cards

NeurobiologyQ?• Match the following sensory receptors with their description.

1. Meissner capsule

NeurobiologyQ?

A. Rapidly adaptive, vibration, receptors 2. Merkel receptors3. Free nerve endings4. Ruffini corpuscles5. Pacinian corpuscles

B. Slowly adaptive, touch and pressure, receptorC. Slowly adaptive, heavy pressure, receptorD. Rapidly adaptive, touch, receptors E. Pain receptors

©™

60

Page 190: Comprehensive Neurology Board Review Flash cards

NeurobiologyA NeurobiologyA.• The correct answers are….• To obtain the answer to this question and to view over 250 more comprehensive neurobiology

questions please purchase the full product here !

BOARD FAVORITE!

questions please purchase the full product here !

Ruffini and Merkel drink TONIC SLOWLY! (slowly adapting tonic receptors)

R M

©™Guyton, Arthur C., John E. Hall. Textbook of medical physiology. W.B. Saunders Company; 10th edition, 2000. p. 521, 530-2

Classification: Neurobiology, Neurophysiology, Touch Receptors

Page 191: Comprehensive Neurology Board Review Flash cards

NeuroanatomyQ?• Which of the following statements is INCORRECT regarding the anatomy of the

cavernous carotid artery?

NeuroanatomyQ?

A. The inferior hypophyseal artery is most commonly a branch of the meningohypophyseal trunk.

B. The inferior hypophyseal artery passes medially to the posterior pituitary capsule.C. Persistent trigeminal arteries can also originate from the posterior vertical

segment of the cavernous internal carotid artery and pass posteriorly through the segment of the cavernous internal carotid artery and pass posteriorly through the posterior wall of the cavernous sinus to join the basilar artery between the origin of the superior and anterior inferior cerebellar arteries.

D. The tentorial artery, or the artery of Bernasconi and Cassinari, is the most inconstant branch of the meningohypophyseal trunk.

©™©™

1

Page 192: Comprehensive Neurology Board Review Flash cards

NeuroanatomyA NeuroanatomyA.• The correct answer is D. The

meningohypophyseal trunk is the most inconsistent branch of the

Meningohypophyseal trunk Inferior hypophyseal artery

meningohypophyseal trunk.• The meningohypophyseal trunk is the most

constant artery (labeled below).

V1

Pituitary

CN3

CerebralPeduncle

CarotidTentorial artery

Dorsal meningeal artery

• The meningohypophyseal trunk usually arises from the posterior aspect of the central third of the posterior bend of the artery at the level of the dorsum sella and frequently gives rise to three branches: the tentorial artery (Bernasconi and Cassinari), th d l i l t d th i f i

V2

V1V1

Middle Fossa

CN5

CN4

©™©™the dorsal meningeal artery, and the inferior hypophyseal artery.

• All other answers are true statements.Isolan G, de Oliveira E, Mattos JP. Microsurgical anatomy of the arterial compartment of the cavernous sinus: analysis of 24 cavernous sinus. ArqNeuropsiquiatr. 2005 Jun;63(2A):259-64

Comments:  Cerebral Arteries, meningohypophyseal, Bernasconi and Cassinari

Page 193: Comprehensive Neurology Board Review Flash cards

NeuroanatomyQ?• α-motor neurons are most commonly found in which Rexed lamina?

A. III

NeuroanatomyQ?

B. VC. VID. IXE. X

©™©™

2

Page 194: Comprehensive Neurology Board Review Flash cards

NeuroanatomyA NeuroanatomyA.• The correct answer is D, IX.• Rexed lamina IX, is located in the ventral motor

f th tt D ib d i 1950 b

BOARD FAVORITE!

areas of the gray matter. Described in 1950s by Bror Rexed as a way to name the different parts of the spinal cord, the Rexed laminae consist of ten layers of grey matter (I-X). It must be noted that Rexed areas are not designated in terms of location but rather by their cellular structure

78

3

1

9

10 9

2

6

45

location, but rather by their cellular structure.• Layer 1 – marginal zone, fast pain (Aδ) and temperature

fibers• Layer 2 – substantia gelatinosa,slow pain (C fibers)• Layer 4 – nucleus propius, interneurons to thalamus• Layer 7 – contains the zona intermedia

L 9 d t

9 9

©™©™• Layer 9 – α and γ motorneurons• Layers are all BOARD FAVORITE topics !Anamizu Y, Seichi A, Tsuzuki N, Nakamura K. Age-related changes in histogram pattern of anterior horn cells in human cervical spinal cord.Neuropathology. 2006 Dec;26(6):533-9.

Comments:  Spine, Rexed lamina

Page 195: Comprehensive Neurology Board Review Flash cards

NeuroanatomyQ?• Match the following cortical

neuroanatomy:

NeuroanatomyQ?

F1. Sylvian fissure2. Rolandic fissure3. Pars triangularis4. Pars opercularis5 P bit li

B C

E

D

GH

5. Pars orbitalis6. Supramarginal gyrus7. Angular gyrus

A

©™©™

36

Page 196: Comprehensive Neurology Board Review Flash cards

NeuroanatomyA NeuroanatomyA.• Cortical neuroanatomy:

1. D, Sylvian fissureMotor cortex Sensory cortex

F

BOARD FAVORITE!

2. E, Rolandic fissure3. B, Pars triangularis4. C, Pars opercularis5. A, Pars orbitalis B C

E

D

GH

6. G, Supramarginal gyrus7. H, Angular gyrus

• Note: F is the precentral sulcus.

A

©™©™

H. Richard Winn, M.D. Youman’s Neurological Surgery 5th Edition. Philadelphia, PA: Elsevier 2004. p. 2533-7.

Comments:  Cortical anatomy, fissures, gyri

Page 197: Comprehensive Neurology Board Review Flash cards

NeuroanatomyQ?• Match the following boundaries of Kawase’s triangle to their anatomic structure.

A. Anteromedial boundary

NeuroanatomyQ?

B. Anterolateral boundaryC. Posterior boundary

1. Greater superficial petrosal nerve (GSPN)2. Internal auditory canal3. Lateral edge of the trigeminal nerve

©™©™

37

Page 198: Comprehensive Neurology Board Review Flash cards

NeuroanatomyA• The correct answers are A - 3, B - 1,

C - 2.• The anteromedial boundary is

NeuroanatomyA.

1• The anteromedial boundary is

formed by the lateral edge of the trigeminal nerve.

• The anterolateral boundary is formed by the greater superficial petrosal nerve (GSPN).

O. C

2 34

5

6

• The posterior boundary is formed by the internal auditory canal or a line connecting the hiatus fallopii to the dural ostium of Meckel’s cave.

1- Anteromedial triangle

2- Medial triangle

3 Superior triangle

4- Lateral triangle (Parkinson's)

5- Posterolateral triangle (Glasscock's)

6 Posteromedial triangle (Kawase's)

©™©™H. Richard Winn, M.D. Youman’s Neurological Surgery 5th Edition. Philadelphia, PA: Elsevier 2004. p. 921-5.

3- Superior triangle 6- Posteromedial triangle (Kawase s)

Comments:  Kawase’s triangle, boundaries, CN V, Greater superficial petrosal nerve

Page 199: Comprehensive Neurology Board Review Flash cards

NeuroanatomyQ? NeuroanatomyQ?• The vertebral arteries enter within the transverse foramina of which vertebrae:

A. C4 to C1B. C6 to C1C. C6 to C2D. C7 to C2E. C8 to C2

©™©™

52

Page 200: Comprehensive Neurology Board Review Flash cards

NeuroanatomyA NeuroanatomyA. BOARD FAVORITE!

• The correct answer is B, C6 to C1.• The vertebral artery passes through the transverse

foramina of C6 through C1 before exiting the transverse

Anterior arch of atlas

Vertebral arterySuperior articularforamina of C6 through C1 before exiting the transverse foramina of the atlas (C1).

• At this location the vertebral artery curves posterior and superior to lie in a groove on the upper surface of the atlas .

• At C7 the vertebral artery swings anteriorly and laterally

ySuperior articular fovea

because it normally runs outside the rudimentary transverse foramina of C7. When a foramina is present, transverse foramina of the seventh cervical vertebra almost always contains vascular and nerve branches as well as fibrous and adipose tissues.

Cranial view of the course of the vertebral arteryover C1

©™©™Katsuta T, Rhoton AL Jr, Matsushima T.The jugular foramen: microsurgical anatomy and operative approaches. Neurosurgery. 1997 Jul;41(1):149-201. MS Jovanovic. A comparative study of the foramen transversarium of the sixth and seventh cervical vertebrae. Volume 12, Number 3 / September, 1990

Comments:  Extracranial Vasculature, Vertebral artery, Cervical spine, transverse foramina 

Page 201: Comprehensive Neurology Board Review Flash cards

NeuroanatomyQ?• Match the structures associated with this portion of

the lateral ventricle:1 Ri ht f f M

NeuroanatomyQ?

A E1. Right foramen of Monro2. Left foramen of Monro3. Choroid plexus4. Thalamostriate vein5. Anterior cerebral artery

A

BF

E

6. Anterior septal vein7. Fornix8. Internal cerebral vein

D

GC

©™©™

D

17

Page 202: Comprehensive Neurology Board Review Flash cards

NeuroanatomyA NeuroanatomyA.• The correct answers are….• To obtain the answer to this question

and to view over 200 more A E

Head of caudate

BOARD FAVORITE!

and to view over 200 more comprehensive neuroanatomy questions please purchase the full product here !

A

B

F

E

D

GC

Thalamus

©™©™

Comments:  Lateral ventricle anatomy, cerebral vascular anatomy